Download as docx, pdf, or txt
Download as docx, pdf, or txt
You are on page 1of 140

Unit –1

Linear Equations and its Applications


1.1 Linear Equations, functions and graphs
1.1.1 Cartesian Coordinates and the Straight Line
Rectangular or Cartesian Coordinates
In a rectangular coordinate system there are two mutually perpendicular reference lines X ' X and
Y ' Y as in the figure. These lines are called the axes of coordinates or coordinate axes, and their

intersection O is called the Origin. The two axes divide the plane in to four quadrants, which are
numbered as in the figure. The coordinate axis X ' X is called the x-axis and Y ' Y is called the y-
axis.

Measuring its distance from each of the coordinate axes along a line parallel to the other axis or
coinciding with it determines the position of a point P.
Distances that are measured to right of axis Y ' Y or up from X ' X are regarded as positive, those
to the left or down as negative. The numerical measures of these distances, with the proper signs,
are called the coordinates of the point; the one representing the distance from Y ' Y is the x-
coordinate or abscissa, and the one representing the distance from X ' X is the y-coordinate or
ordinate. Thus in the figure the abscissa of P is BP =+ 4 , and the ordinate of P is AP=−3 .
The content of this paragraph may now be stated compactly as follows:

pg. 1 Prepared by: Firehun A


Definition:
The abscissa or x-coordinate of a point is the distance from the y-axis to the
point. The ordinate or y-coordinate of a point is the distance from the x- axis
to the point.
In naming a point by its coordinates, we write them in parentheses, putting the abscissa first.
Thus, in the figure P is the point (4, -3). If the coordinates are variable or unknown, the abscissa
is denoted by the letter x and the ordinate by y.
Plotting of Points
It is clear that any point in a plane is fixed by means of its coordinates, for the abscissa locates it
on a parallel to the y-axis and the ordinate on a parallel to the x-axis and these meets in one point.

To locate a point whose coordinates are given [or as commonly stated, to plot the point], first
mark the coordinate axes and choose a unit of measure; then measure off the abscissa on the x-
axis and the ordinate from the end of the abscissa. Thus, to plot (-3, 2), count 3 units to the left
on the x-axis and 2 units up. This is especially convenient when using coordinate paper, i.e.,
paper ruled with sets of equally spaced parallel lines.
The system of coordinates, which has been described, is the rectangular system and is the
particular case of Cartesian coordinates, which is usually employed. In the general Cartesian
system, the axes are not necessarily perpendicular but may be oblique.

pg. 2 Prepared by: Firehun A


Inclination and Slope
If the line is parallel to the x-axis, its inclination is zero. By the well known theorem of
geometry, the angle of inclination of a line is also its angle of intersection with any line parallel
to the x-axis.

Definition:
The Inclination of a line is its angle of intersection with the x-axis.

In this Module the inclination of a line is denoted by the Greek letter  [alpha], and the slope by
m. Thus m = tan . When m is positive,  is obtuse and the line extends downward to the right.

The slope fixes the direction of a line. Hence a line is determined when its slope and one point on
it are known.

Definition:
The slope of a line is tangent to its inclination

The slope of a straight Line


P1 ( x 1 , y 1 )
The slope of a line passing through the points to
P2 ( x 2 , y 2 )
is given by the formula:
y 1− y2
m=
x 1 −x 2

pg. 3 Prepared by: Firehun A


Proof:
MP 1
tan α =
In both figures P2 M

Since the projections of a segment P1 P2 on the x-axis or any line parallel to it is x 1−x 2 and the

projections of a segment P1 P2 on the x-axis or any line parallel to it is y 1 − y 2 , it is possible to

have P2 M = x 1−x 2 and MP 1 = y 1 − y 2 .


Substituting this leads to have a formula
y 1− y2
m=
x 1 −x 2 .

Now then you can apply the slope formula to find the slope and inclination of a line through two
given points and to draw a line through a given point with a given slope.
Parallelism and Perpendicularity
If two lines are parallel, then their slopes are equal, and conversely. If they
are perpendicular, then their slopes are negative reciprocals, and
conversely.

Proof:

pg. 4 Prepared by: Firehun A


If l 1 and l 2 are parallel, obviously α 1=α 2 ; hence the slopes are equal.

If l 1 is perpendicular tol 2 , draw through the intersection of l 1 and l 2 a line parallel to the x-axis;
0 0
then the inclination α 1=α 2 +90 . Since tan(α+90 )=−cot α by trigonometry,
1
tan α 1 =tan( α 2 +900 )=−cot α 2=−
tan α 2 .

1
m1 =−
Substitution of m 1 and m 2 gives m2 or m 1⋅m2 =−1

Summarizing the above theorem will enable you to state the following two tests.
Test for parallelism:
m 1 =m 2
Test for perpendicularity:
m 1⋅m2 =−1
1.1.2. The Straight Line
The preceding sub section is a general treatment of the fundamental types of problems, which
arises, in coordinate geometry. I begin this subsection with the straight line, for which various
properties, such as its determination either by two points or by a point and a direction, lead to
relations between the coordinates of its tracing point that can be expressed in the form of an
equation. There are several forms of the straight-line equation, but all are of the first degree and
for a particular line each one may in general be reduced to any of the others.

pg. 5 Prepared by: Firehun A


The Point-Slope and Two Point Forms

Let the line l be fixed by a point and a direction. Let the point be P1 ( x 1 , y 1 ) and let the direction

of l be given by the slope m = tan. Let P( x , y ) be any other point on l.

y− y 1
m=
By the slope formula, x−x 1 or:

y− y 1=m( x−x 1 )

That is, the coordinates of every point on l, including those of P1 ( x 1 , y 1 ) , satisfy equation
y− y 1=m( x−x 1 ) . Conversely, let P2 (x 2 , y 2 ) be any point other than P1 ( x 1 , y 1 ) whose

coordinates satisfy y− y 1=m( x−x 1 ) . Then


y 2− y 1
m=
y 2 − y 1 =m( x 2 −x 1 ) or x 2 −x 1 .

Hence the slope of P1 P2 is m and P2 lies on l. Therefore y− y 1=m( x−x 1 ) is the equation of
the line l. This is called the point-slope form of the straight-line equation. It is used in writing
the equation of a line when one point and the slope are given.
The point-slope form may also be used when two points are known. For if the line is determined
y 1− y 2
m=
by the points P1 and P2 , then by the slope formula x 1 −x 2 . Substituting of this in

y− y 1=m( x−x 1 ) gives:


y 1− y 2
y− y 1=
x 1 −x2 ( x−x1 )

The last equation is known as the two-point form of the straight-line equation.

pg. 6 Prepared by: Firehun A


The Slope-Intercept Form
If the slope and the y-intercept are given, the line is determined by a point on the y-axis, (0, b)

and the slope m. Substituting in y− y 1=m( x−x 1 ) , you have:


y−b=m( x−0) or
y=mx+b , which is called the slope-intercept form of the straight-line equation.
This form can be useful in finding the equation of a straight line when the slope and the y-
intercept are known. It’s most important use is in finding the slope of a line from its equation.

For, if the equation of the line is reduced to the form y=mx+b by solving for y, the coefficient
of x is then the slope of the line and the constant term is its y-intercept.
The Intercept Form
Suppose that the intercepts of the line are given; let the x-intercept be a and the y-intercept b.

b
m=−
Here the line is determined by two points (a, 0) and (0, b). You have at once, a . Then by
b
y−b=− ( x−0 )
the point-slope form, a , which reduces to:
x y
+ =1
a b This is called the intercept form of the straight-line equation. It is used in writing the
equation of a straight line when the intercepts are known.
Lines Parallel to the Axes
The equation of a line parallel to the y-axis can not be written in any of the forms so far given
since there is nether a y-intercept nor a slope.

pg. 7 Prepared by: Firehun A


The equation of such a line is obviously:
x=a

Similarly, the equation of a line parallel to the x-axis has the form:
y=b
Example: Find the equation of the line, say line l, through (8, 3) that is perpendicular to the line

say line m, with equation4 x− y=5 .

You first write the equation of line m in the slope-intercept form, y=4 x +5 .
1

Hence line m has slope 4. The perpendicular line l must have slope 4 . So the equation of line l
is:
1
y−3=− ( x−8)
4 .
1
y=− x +5
So, the equation of l is 4
The preceding discussion shows that the equation of any straight line is a first-degree equation;

that is, an equation that can be reduced to the form Ax+By+c=0 for some non zero constant

A, B, and any real number C. Conversely, the graph of the first degree equation Ax+By+c=0
is a straight-line.
1.2 Applications of linear equations
1.2.1 Application in Business and management
Now, you have completed studying the four forms of linear equations. If you have properly
understood them, then you will come across with application problems, which are completely
dependent on one or more of those forms of linear equations.
Example: The fixed cost for manufacturing a particular stereo system is $96000 and the variable
cost per unit is $80. If x units are manufactured,
a. Express the cost function as function of x; and
b. Find the cost of producing 500 stereos.
Let fixed cost = b = $96000, and variable cost = m = $80
a. This is an application problem on slope–intercept form, that is, since we are given slope and
y–intercept, we can apply the slope–intercept form of linear equation to solve the above

pg. 8 Prepared by: Firehun A


application problem. Hence, the cost function is given by the slope–intercept form: c(x) = mx
+ b and substitute for m and b as shown below, which is the required cost function:
C(x) = 80x + 96000
b. To determine the cost of producing 500 stereo systems use the above obtained cost function
and substitute 500 for x which is known to be the cost of producing 500 stereos. That is, the
cost of producing 500 stereo systems:
c (500) = 8 x 500 + 96000
= 8 x 500 + 96000
= 4000 + 96000
Hence, the cost of producing 500 stereos = $100,000

Cost, Revenue and Profit Analysis

Definitions:
1. The function, c, which is defined by c(x) = mx + b, where m is
variable cost, b is the fixed cost and x is the volume of output, is
called the cost function.
2. The function, r, which defined by r(x) = px, where p is the rate of
selling price per unit item and x is the total volume of output, is
called the revenue function.
3. The function, p, which is defined by p(x) = r(x) – c(x) is called the
profit function.

So, what do you think will be cost, revenue, and profit relationships? And what assumptions do
you consider about the behavior of cost and revenue?
Any analysis, whether graphical or algebraic, makes certain assumptions about the behavior of
costs and revenue. Cost-volume-profit analysis makes the following assumptions:

pg. 9 Prepared by: Firehun A


a. Revenue per unit of output is constant. Thus total revenue varies directly with volume.
b. Costs can be divided into either fixed or variable.
c. Fixed costs are those remain constant over the time period considered for all levels of output.
d. Variable costs are costs that are constant per unit of output regardless of volume and thus
fluctuate in total amount as volume fluctuates.

Depending on these assumptions, the behavior of revenue and cost functions can be represented
graphically by straight-line diagram as shown below.

Revenue Behavior Cost Behavior

1.2.2 Break–even Analysis


The most popular approach to cost-volume-profit analysis is referred to as break-even analysis.
The break-even approach focuses to the profitability of the business and identifies the level of
output at which the business neither makes a profit nor sustains a loss, that is, the net income is
equal to zero, which is known to be the break-even point.

Since net income is zero, the break-even point is the level of output at which

pg. 10 Prepared by: Firehun A


Net Income = Total Revenue – Total Cost = 0
That implies,
Total Revenue = Total Cost.
The graph of the revenue behavior and the cost behavior on the same X–Y plane, which is shown
in the next figure, is called the break-even chart.

Note That:
 The horizontal axis is used to represent volume of output either as number or units or as a
percent of capacity. The vertical axis represents Br. values. The origin is at zero-
representing zero volume and zero Br.
 The revenue line is drawn by plotting two or more total revenue points, one of which is
always the origin, and joining them using a straight line.
 The fixed cost line is drawn parallel to the horizontal axis from point on the vertical axis,
which represents total fixed cost in Br.
 The total cost line is drawn by plotting two or more total cost points (one of which is always
the point were the fixed cost line starts on the vertical axis) and joining them using a straight
line.
 The point where the total revenue line and the total cost line intersect is the break-even point.
 The point of intersection of the line drawn perpendicularly from the break-even point to the
horizontal axis and the horizontal axis indicates the break-even volume [or break even
output] in units or as a percent of capacity.
 The point of intersection of the line drawn perpendicularly from the break-even point to the
vertical axis and the vertical line indicates the break-even volume in Br.

pg. 11 Prepared by: Firehun A


 The area between the horizontal axis and the fixed cost line represents the fixed cost in Br.
 The area between the fixed cost line and the total cost line represents the total variable cost in
Br. for the various levels of operations.
 The area between the total revenue line and the total cost line to the left of the break-even
point represents the loss area where total revenue is less than total cost.
 The area between the total cost line and the total revenue line to the right of the break-even
point represents the profit area where total revenue is greater than total cost.
Now, let me give you an example in relation to our discussion above.
Example: A publisher is planning to produce a new textbook. The fixed costs per day are Br.
240, variable cost is Br. 20 per book, and the selling price is Br. 35 per book. Draw the break-
even chart and indicate the break-even point.
Now, try to follow the solution of the above problem seriously so that you will be able to solve
similar problems by yourself.
Let x = total output of textbooks.
Fixed cost = b = Br. 240
Variable cost = m = Br. 20
Selling price = p = Br. 35
⇒ Total Revenue = r(x) = px

= 35x
⇒ The total Cost = c(x) = mx + b

= 20x + 240
Since a break-even point is the point where total cost and total revenue are equal, we have
r(x) = c(x)
⇒ 35x = 20x + 240.

Solving the above equation for x gives:


35x – 20x – 240 = 0
15x = 240
x = 240/15 = 16 textbooks.

The publisher will get no profit or no loss when it produces and sells 16 textbooks. Thus the
point that shows the relationship between the this level of production the publisher’s total cost or

pg. 12 Prepared by: Firehun A


this same level of production and the publisher’s total revenue, (16, Br. 560), is known as the
break-even point.

The break even chart was constructed in the rectangular coordinate system for some chosen
values of x and the corresponding cost and revenue values, as follows:
The break-even chart

Example: A company is planning to manufacture a certain product. After conducting a survey,


the researching department provides the following estimates; a weekly demand of 200 items at a
price of Br. 16 per each item produced, and a weekly demand of 300 items at a price of Br. 14
per each item produced. The financial department estimates that a weekly fixed cost will be Br.
1400 and the variable cost per unit item produced will be Br. 4.
Now let us see how it is possible to formulate mathematical models that fit the Demand,
Revenue and Cost.
The demand equation can be obtained on the basis of the Two Point Form of equations of a line.
That is a straight line can be fitted by a linear equation whenever two distinict points are given

along the line. Let (a, b) and (c , d ) be two different points along a certain line. Then the
equation of the line will be obatined by using the Two Point Form of equation of a line as:
( x−a )⋅(d−b )=( y−b )⋅( c−a )
Thus, an equation that can determines the Demand can also be obtained by making two different
points, which are constructed on the basis of the unit price and the total demand of the product.

That is, (200 , 16 ) and (300 , 14 ) .


Applying the above form yields;

pg. 13 Prepared by: Firehun A


( x−200 )⋅(14−16 )=( y−16 )⋅(300−200)
Transforming this equation to its lowest form gives:
1
y =20− x,
50
1
⇒ p (x )=20− x
50 [1]
Where x is the total level of production and p (x) is the price per unit item when the total deman x
is given.
Since Revenue is defined as the amount of money recieved by the company for selling x units of
items at a price Br. p per unit, we do fit the revenue by the equation:
r( x )=px , [2]
Where p is the selling price per unit item and x is the total demand or the total level of production
that should be made and sold.
Thus, replacing p of of Equation [2] by the corresponding expression in Equation [1], it is
possible to fit the Revenue by a quadratic equation of the form:
r ( x )=( p )⋅x
1 2
r ( x )=20 x− x
50
Similarly, an equation that fits the Cost can be obtained by making two different points, which
show the relationship between number of items produced and the total cost spend in producing
that number of items.

That is, ( 0 , 1400 ) and(1 , 1400+4 ) .


Applying the Two Point Form of equation of a line yields;
( x−0 )⋅( 1404−1400 )= ( y−1400 )⋅(1−0 )
Transforming this equation to its lowest form gives:
y=4 x+1400
c ( x )=4 x +1400 ,
Where x is the total level of production and c (x) is the total cost of production when x is given.If
you are capable of answering all the questions, please proceed further. If you find some of them
difficult to answer, you need to go back and revise.

pg. 14 Prepared by: Firehun A


1.2.3 Rational functions its applications and Market Equilibrium Analysis
Let us consider the concept of rational functions before defining what average cost, profit and
rate of profit mean. Rational functions are defined as fractions involving polynomial functions or
as quotients of polynomial functions. Although the idea of a fraction is comparatively simple, it
has proved throughout the ages a somewhat difficult one to grasp. A fraction is a device, which
represents our thinking of something as broken in to a number of parts, and then considering a
certain number of those parts. In the forthcoming discussion you will be learning three more
mathematical models: the rational mathematical models, average cost, average profit and rate of
profit.
Now let me use the idea of fractions or rational numbers to give you a brief explanation on the
concept of rational functions. The fraction is written with a horizontal or slanting line segment

(
between the numbers b
a
or
a
b
, b≠0 ); the two numbers are called the terms of the fractions.
The lower number is called the denominator and the upper number is called the numerator.

Similarly, if f ( x ) and g( x) are two polynomials such that g( x )≠0 , then it is possible to

formulate a fraction or rational expression that involves the two polynomials, f ( x ) as a

numerator and g( x )≠0 as a denominator.


Let me now give you the definition of rational functions of one variable x.

Definition:
Suppose that p and q are polynomial functions, where q is not the zero polynomial
p( x)
f ( x )=
function. Then the function f given by the equation q( x ) is called a Rational
Function.
The domain of the function is the set of all real numbers x for whichq ( x )≠0 .

If p( x) and q ( x) have no common factors, then the function f is said to be in reduced


form or lowest term.

pg. 15 Prepared by: Firehun A


Following are examples of rational expressions.
x3 +2 x2 −1 2 x 2 −4 x
3
a. x +2 b. x −1
2 x 2 −x−1 1
c. 1 d. x−2

Average Cost and Average Profit


The rational function, defined by cost per unit, is called average cost. And it is defined as
follows.
Definition:
If x is number of units of item produced in some time interval, then average
c( x)
c ( x )=
cost, c( x) , will be defined by x , which is known as cost per unit.

The function p(x) is average profit function if p(x) is profit per unit and defined as follows.

Definition:
If x is number of units of item produced in some time interval, then the
p( x )
p( x )=
rational function, p( x) , which will be defined by x , is called the
average profit [or profit per unit].

Example: A factory has a fixed daily overhead cost of Br. 600. If it produces x units daily, then
the cost for labor and materials is Br. 3x. The daily cost of equipment maintenance is Br.
x2
240000
a. Find a function giving the total daily cost, c(x), when x units are produced.
b. Find the average cost of producing x units of items.
c. What is the average cost of producing 500 items?
a. Since a fixed cost Br. 600, Labor and material cost Br. 3x, and equipment maintenance cost
x2
Br. 240000 , a function giving the total daily cost, c, will be obtained as a function of x as:

pg. 16 Prepared by: Firehun A


x2
c(x) = 240000 + 3x + 600
c( x)
c ( x )=
b. Since the average cost is computed by the formula x

c ( x )=
[x2
240000
+3 x+600
=
x ]
+3+
600
x 240000 x
( 500 ) 600
c (500 )= +3+ =4 . 2021
c. 240000 ( 500 ) .
The average cost per unit item in the production of 500 units is obtained to be Br. 4.20

Rate of Profit
The rational function which is usually explained as the ratio of profit function to cost function is
called the rate of profit. Formally, the definition of rate profit is given as follows:

Definition: Let p(x), c(x), and r(x) be the profit, cost, revenue functions,
respectively. Then, the Rate of Profit, as a rational function, is defined by the

equation:
RoP( x )=
[ ]
p( x )
c( x)
×100 %
.Since p( x)=r( x)−c( x) , RoP( x ) will be

reduced in to its lowest form as


RoP( x )= [ r( x )
c( x) ]
−1 ×100 %

Now, let me do the following example to illustrate (explain) the above three mathematical
models. I request you to follow carefully the solutions below the questions.
Example: Given the total cost function in Birr of producing x units of items is
x2
c ( x )=1000+ 100 x −
4 and the total revenue in Birr isr( x )=100 x . Find:
a. the profit function;
b. profit obtained by producing 100 items;
c. average cost function;
d. average cost per unit if 100 units of items are produced; and

pg. 17 Prepared by: Firehun A


e. average profit of a unit of product when 100 units are produced.

a. Since p( x)=r( x)−c( x) , we have,

p( x)= 100 x - [ 1000+100 x−


x2
4 ]
= 4[
x2
−1000 ]
b. p(100 )= 4 [
(100)2
−1000 =1500 ]
c ( x )=
[
1000+100 x−
x2
4 x
= +100+
1000 ]
c. x 4 x f
d. Br. 85

[ x2
4
−1000 ]
x 1000
= −
e. Since p( x)= x 4 x , we would have the average profit per unit of items to
be Br. 15.

If you are capable of answering the entire question, please proceed further. If you find some of
them difficult to answer, you need to go back and revise.

pg. 18 Prepared by: Firehun A


pg. 19 Prepared by: Firehun A
UNIT -2
Matrix Algebra and its Applications
2.1 Review on Matrix Algebra
This section will introduce you to the theory of matrices, in particular to the meaning of matrix
and matrix algebra. Without matrix algebra, linear programming and some important solutions
of real world problems can be impractical because of large number of variables and a number of
linear equations.
Before defining what matrix is please go through the following illustrative examples:

[ ]
1 0 4
1. 3 2 5 is a 2¿ 3 matrix.

2.
[ 16 −35 ] is a 2¿ 2 square matrix.

[ ]
2 4
1 8
3. 0 6 is a 3¿ 2 matrix.

[]
1
1
4.
3 is a 3¿ 1 column matrix [vector].

5. [ 1 2 3 ] is a 1¿ 3 row matrix [vector].

[ ]
1 0 0
0 1 0
6.
0 0 1 is a 3¿ 3 identity matrix.

[
0 0 0
]
7. 0 0 0 is a 2¿ 3 zero matrix.
In line with the above illustrative examples, the following are the definitions matrix and other
related definitions to matrices.

pg. 20 Prepared by: Firehun A


Definition:
 Matrix is an m¿ n rectangular array of numbers.
 Each number of the m¿ n matrix is called an element or entry of the m¿ n matrix, where
m represents the number of rows and n represents the number of columns of the m¿ n
matrix.
 A matrix with one row is called row vector (a row matrix).
 A matrix with one column is called a column matrix (column vector).
 Their orders or shapes or sizes or dimensions can classify matrices.
 m¿ n is the order or shape or size or dimension of m¿ n matrix.

Consider the following matrices:

[ ]
1 3 2
5 8 2
A = 8 7 0 is a 3 x 3 matrix.

[ ]
0 3 7
4 9 2
B=
−1 4 9 is a 3 x 3 matrix.

[ ]
1 3 2
5 8 2
C = 8 7 0 is a 3 x 3 matrix.

D=
[ 13 0 4
]
2 5 is a 2 x 3 matrix.

E=
[3
1 0 5−1
]
4 /2 5 is a 2 x 3 matrix.
What do you say about the above matrices?
Matrices A, B and C have the same size (shape) and are known to be square matrices. Not only
that A and C, D and E have the same shape and corresponding elements, they are also known to
be equal matrices.
So, can you guess the definition of square matrices and equal matrices?

pg. 21 Prepared by: Firehun A


2.2 Matrix operations and Techniques
Addition of Matrices
The sum two or more matrices of the same size can be obtained by adding the corresponding
elements of two or more matrices. This is possible only if the matrices have the same shape;
otherwise the sum of the two or more matrices is undefined.

Definition
The sum of two or more m x n matrices is obtained by adding corresponding
elements of the two or more matrices only if the two or more matrices have the
same order.

[ ] [ ]
2 5 5 7

Example: Let A =
4 8
1 0 , B=
1 3 2 1 8
2 6 and C = 2 3 6 [ ]
Find: a. A + B = b. A + C

[ ][ ][ ][ ]
2 5 5 7 2+5 5+7 7 12
4 8 1 3 4+1 8+3 5 11
a. A + B = 1 0 + 2 6 = 1+2 0+6 = 3 6
b. Matrix A, and matrix C cannot be added.
This is due to the fact that A is a 3 x 2 matrix, but C is a 2 x 3 matrix, i.e. matrix A and C does
not have the same order or size.

Properties of addition of matrices:


For any four matrices A, B, C and O
 A + O = O + A = A if and only if O is the additive identity
matrix;
 A + B = B + A = O if and only if A is the additive inverse of B
and vice versa;
 A + B = B + A, addition of matrices is commutative; and
 (A + B) + C = A + (B + C), addition of matrices is associative.

pg. 22 Prepared by: Firehun A


Please go through the following Checklist and for the part, which your answer is “I can’t” go
back to the part and revise it thoroughly.
Multiplication of Matrices
Scalar Product

Definition
The product between a real number and a matrix is called scalar
product.

[ ]
1 8 4
4 9 5
5 5 8
Example: Let k = 4 and A =
2 2 0 Find k ¿ A.

[ ][ ]
1 8 4 4 32 16
4 9 5 16 36 20
5 5 8 20 20 32
k ¿A = 4 ¿
2 2 0 = 8 8 0
Dot product

Definition:
The product between a 1 x n row vector and n x 1 column vector is
called dot product.

Example: Let A = [ 2 1 0 3 6]

[]
3
1
2
0
B=
5
Find the dot product of A and B.

pg. 23 Prepared by: Firehun A


[]
3
1
2
0
A x B =[ 2 1 0 3 6 ] x
5
=2x3+1x1+0x2+3x0+6x5
= 6+1+0+0+30
= 37
Note: The result of a dot product is a real number (constant)

Example: A factory produces shoes that require 4 labor-hours per pair of shoes is the fabricating
department and 1 labor hour in the finishing department. Fabricating department personnel
receives Birr 8 per hour and finishing personnel receives Birr 6 per hour. Find total labor cost per
pair of shoes.

Let A = labor hours = [ 4 1]


8
B = labor cost = 6
[]
Total labor cost = A x B

= [4 x
[
1] 6]
8
= Br. (4 x 8 + 1 x 6) =Br. 38.
Product of Matrices
Consider two matrices, A and B. And let the number of rows of matrix B is the same as the
number of column of matrix A.

Definition:
Product of matrices A and B, AB, is defined only if number of rows B is the
same as number of columns of A and the order of the resulting matrix is the
same as number of rows of matrix A and number of columns of matrix B.
If A is an m x n matrix and B is an n x p matrix, then AB is an m x p matrix.

pg. 24 Prepared by: Firehun A


[ ]
1 4 2
2 1 8
Example: Let A = 4 5 3 , B =
[
7 −5 −2
0 1 2 ]
Find a. AB; and b. AB.

][ ]
1 4 2

a. AB = 4 5 3
[
2 1 8 7 −5 −2
0 1 2

[2 x 1+1 x 7+8 x 0 2 x 4 +1 x -5+ 8 x 2 2 x 2+1 x -2+8 x 2


= 4 x 1+5 x 7+3 x 0 4 x 4 +5 x-5+3 x 1 4 x 2 +5 x -2+3 x 2 ]
=
[ 4 +35+ 0 16+−25+3
2 + 7 + 0 8+ -5+16 4 +-2+16
8+-10+6 ]
=
[ 399 −611 184 ]
b. BA is not defined, since the number of columns of B is not equal to the number of rows of A.
Example: The personnel computer retail company sells five different computer models through
three stores located in a large metropolitan area. The inventory of each model on hand in each
store is summarized in matrix A. Wholesale (W) and retail (R) values of each model computers
are summarized in matrix B.

A B C D E
4 2 3 7 1 Store I
A=2 3 5 0 6 Store II
10 4 3 4 3 Store III

$700 $840 A
$1400 $1800 B
B = $1800 $2400 C
$2700 $3300 D
$3300 $4900 E
a. What is the retail value of the inventory at store II?
b. What is the wholesale value of the inventory at store III?

pg. 25 Prepared by: Firehun A


840
1800
a. The retail value of the inventory at store II = 2 3 5 0 6 2400
3300
4900

= $(2 x 840 + 3 x 1800 + 5 x 2400 + 0 x 3300 + 6 x 4900)


= $(1680 + 5400 + 12000 + 29400)
= $48480
700
1400
b. The whole value of inventory at store III = 10 4 3 4 3 1800
2700
3300

= $(10 x 700 + 4 x 1400 + 3 x 1800 + 4 x 2700 + 3 x 300)


= $(7000 + 5600 + 5400 + 10800 + 9900)
= $ 38700

Elementary Row Operations


There are three fundamental elementary row operations, which may be performed on any row or
rows of any given matrix. Elementary row operations are important in the solution of problem,
like linear programming problems, solving systems of linear equations etc.
The three elementary row operations can be defined as follows:

Definition: The First Elementary Row Operation


The first elementary row operation is an operation in which any two rows of a

matrix are interchanged, i.e.,


Ri ↔ R j
The arrow means “replace.”

pg. 26 Prepared by: Firehun A


Example: Perform the first elementary operation on the second and the third rows of the
following matrix:

[ ]
2 3 1
7 0 −2
−3 6 1
A=
0 9 11

[ ] [ ]
2 3 1 2 3 1
7 0 −2 −3 6 1
−3 6 1 7 0 −2
0 9 11 R2 ↔ R3 0 9 11

Definition: The Second Elementary Row Operation


The second elementary row operation is an operation in which
elements of a row are multiplied by a non-zero constant (k) and then
replaces the original element of that particular matrix by the resulting

elements; i.e.,
kR j →R j
.

[ ]
2 7
0 4
Example: Let k = 4 and A = 1 8 be given. Use the second elementary row operation on the

third row elements, i.e.,


kR j →R j .

[ ] [ ]
2 7 2 7
0 4 0 4
A=
1 8 4 R3 → R3 4 32

Definition: The Third Elementary Row Operation


The third elementary row operation
2 7 is an operation in which a constant
(k) multiple of another row elements is added to a given row elements
and then replaces the original row elements by the respective result. i.e.,
Ri + kR j → Ri

pg. 27 Prepared by: Firehun A


2 4
[ ]
Example: Let k = 3 and A = 1 8 . Perform the third elementary row operation on the second
row by adding 3 time elements of the first row to the second row’s elements. i.e.,
3 R1 + R 2 → R2
.

A=
[ 21 48 ] 3 R1 + R 2 → R2 [ 27 204 ]
1.1.4. The Augmented Matrix
What is Augmented Matrix ?

Definition:
The combination of two or more matrices written as one is called
augmented matrix. The vertical bar separates the two original matrices.

Example: Let A =
[ 21 48 ] and B =
[ 47 38 ] , then the augmented matrix

[ A|B ] = [ 2 44 3
|
1 87 8 ] .
2.4 The Inverse of a matrix

In ordinary algebra, we write a ( a ) =( a 1 ) ( a−1 )=a1−1 =a 0=1


−1

−1
Thus, a xa = 1.
−1 −1
And a is called the multiplicative inverse of a because the product of a and a is the
multiplicative identity, 1.

In matrix algebra we have an analogous operation illustrated by

[ ][
1 2 −3 2
=
1 0
2 3 2 −1 0 1
=I
][ ]
That is, the product of the two matrices at the left is the identity matrix I of order 2. If we call the
leftmost matrix A, that is,

pg. 28 Prepared by: Firehun A


1 2
A= 2 3
[ ]
−1
We define the second matrix as the multiplicative inverse, A , and we write

A −1
=
[−32 −12 ] Thus, we have
A⋅A−1 =I

Definition:
Two square matrices are inverses of each other if their product is the identity
matrix, I.

Note that the definition says inverses of each other. This means that either matrix can be called A
−1 −1 −1
and the other is A or, more to the point, it is possible to write the product as A A or A A
and still obtain the identity matrix I. Hence, any two square matrices that are inverses of one
another will satisfy the rule of commutatively of multiplication.
For the above example it is right to say:

[−32 −12 ][ 12 23 ]=[ 10 01 ]=I


We shall see later that not every matrix has an inverse. However, if the inverse of A exists, it is
unique: that is, a matrix that has an inverse has exactly one inverse. We are now ready to attack
one of the objectives of this section, which to compute the inverse of a given square matrix, if
one exists.

Gauss-Jordan Inversion
Here I will present the method attributed to Gauss and Jordan, which is the method we shall use
as we continue our work.
Briefly, the Gauss-Jordan method starts by writing the given matrix at the left and the
corresponding identity matrix on the right as an augmented matrix. Then select and carry out
elementary row operations that will convert the given matrix in to the identity matrix, and apply
the same operations to the matrix at the right. When the left [given] matrix becomes identity

pg. 29 Prepared by: Firehun A


matrix, the matrix on the right [in the augmented matrix] will be converted into the desired
inverse. Let us consider the following example to illustrate the above concept.
2 4
Example: Find the inverse A-1, if A = 1 −1
[ ]
[ A|I ] = [ 2 4 1 0
|
1 −1 0 1 ] R1 ↔ R2 [ 1 −1 0 1
|
2 4 1 0 ]
−2 R1 + R 2 → R2 [ 1 −1 0 1
|
0 6 1 −2 ]
1
6
R 2 → R2 |
[
1 −1 0 1
0 1 1/6 −1/3 ]
R1 + R 2 → R1 [10 0 |1/6 2/3
1 1/6 −1/3 ][
= I|B ]

Hence, B = A
−1 [
1/6 2/3
= 1/6 −1/3
]
2. Find A-1. If A =
[ 21 −4−2 ]
.

[ A|I ] = [ 2 −4 1 0
|
1 −2 0 1 ] R1 ↔ R2 [ 1 −2 0 1
|
2 −4 1 0 ]
−2 R1 + R 2 → R2 [ 1 −1 | 0 1
0 0 1 −2 ]
Since elements of the second row to the left of the bar are all zero, A has no inverse. That is, the
inverse of matrix A does not exist.

2.5 Applications Of Matrix


Dear distance learner, in this Section you will study how to use the Gauss Jordan method to solve
m x n linear systems of equations wherem≠ n . You have developed the idea of applying
elementary row operations on augmented matrices and now you will consider these operations
are applied to solve linear systems equations involving more than two variables.

pg. 30 Prepared by: Firehun A


2.5.1 Method of Solving System of Linear Equations
2.5.1.1 Gauss – Jordan Method
Solution of system of linear equations is referred to as Gauss Jordan Elimination. We will use
this method to solve larger-scale system in the next section, including systems where the number
of equations and the number of variables are not the same. For ease in understanding, we first

present the case where the number of rows m is greater than the number of columns n.

Definition:
Solution of system of linear equations based on elementary row
operations and augmented matrix is called Gauss Jordan method of
solving system of linear equations

Consider the 3 by 2 system


4 x1 +5 x2 =30
3 x 1 +2 x2 =19
2 x1 +5 x2 =20
Proceeding in the usual manner, we start with matrix

[ ] [ ]
4 5 30 1 5/4 15 /2
3 2 |19 R1 3 2 |19
2 5 20 ↔ R1 2 5 20
4

[ ]
1 5/4 15/2
0 −7/4 |−7/2
3 R1 + R 2 → R2 , −2 R1 + R 3 → R3 0 5/2 5

[ ]
1 5/ 4 15 /2
4 0 1 | 2
− ×R 2 ↔ R 2 0 5/2 5
7

pg. 31 Prepared by: Firehun A


[ ]
1 0 5
5 5 0 1 |2
− R2 + R 1 → R1 , − R 2 + R3 →R 3 0 0 0
4 2
At this point, the method terminates since there is no entry in the third row and third column on
the left hand side of the matrix. The first row of this matrix tells
x 1=5
The second row,
x 2 =2
and the third row,
0=0
Thus our solution is given by
x 1=5 and x 2 =2 .

Now, consider the 3 by 2 system


4 x1 +5 x2 =30
3 x 1 +2 x2 =19
2 x1 +5 x2 =20
Here we start with
Proceeding in the usual manner, we start with matrix

[ ] [ ]
4 5 30 1 5/4 15/2
3 2 |19 R1 3 2 |19
2 5 30 ↔ R1 2 5 30
4

[ ]
1 5/4 15/2
0 −7/4 |−7/2
3 R1 + R 2 → R2 , −2 R1 + R 3 → R3 0 5/2 15

[ ]
1 5/ 4 15 /2
4 0 1 | 2
− ×R 2 ↔ R 2 0 5/2 15
7

[ ]
1 0 5
5 5 0 1| 2
− R2 + R 1 → R1 , − R 2 + R3 →R 3 0 0 10
4 2

pg. 32 Prepared by: Firehun A


Again the method terminates, but this time the final matrix gives
x 1=5 , x 2 =2 , and 0 = 10.

Of course, the statement 0 = 10 is absurd, so that the system has no solution. By the way, the

result above still tells us something about the system. Indeed, x 1=5 and x 2 =2 is a solution to
the first two equations of the system but not the third. How much is the third equation off by? If
you guessed precisely the 10 in the last absurd statement, 0 = 10, you were correct. Lastly, it is
possible but rare that a 3 by 2 system [in general, m by n with m > n] will have an unlimited
number of solutions. The Gauss-Jordan method will identify this situation, as indicated by the
following summary of all three possibilities.

To solve an m x n system of equations with m > n, we start with the


augmented matrix

[ A|b ]
and attempt to transform it in to the matrix

[ I|s ]

One of three things will result:


1. An n by n identity matrix above m – n bottom rows that are all zeros,
giving a unique solution; example:

[ ]
1 0 0 5
0 1 0 −2
0 0 1| 5
0 0 0 0
0 0 0 0
2. A row that is all zeros except in the constant column, indicating that
there are no solutions; example:

[ ]
1 0 0 5
0 1 0 −2
0 0 1| 5
0 0 0 2
0 0 0 0
pg. 33 Prepared by: Firehun A
3. A matrix in a form different from [1] and [2], indicating that there are
an unlimited number of solutions; example:
[ ]
1 0 2 5
0 1 3 −2
0 0 0| 0
0 0 0 0
0 0 0 0

Now let’s consider the case where the number of rows m is less than the number f columns n;
that is, m < n. This time the system has either no solutions or else an unlimited number of
solutions. There cannot be just one column solution

Our attempt to transform [ A|b ] into [ I|s ] in the case where m < n will result in:
1. A row which is all zeros, except in the constant column, indicating that there are no
solution; or
2. A matrix in a form different from [1], indicating that there are an unlimited number of
solutions.

Let us look at an example.


Example: Find the solution of the linear systems of equation:
2 x+2 y + z=36
x+3 y+2 z=30
Starting with the augmented matrix

[ 2 2 1 |36
1 3 2 30 ]
we interchange rows 1 and 2, obtaining

[ 1 3 2 30
|
2 2 1 36 ]
and then add -2 times row 1 to row 2:

[ 1 3 2 30
|
0 −4 −3 −24 ]
Next, we divide row 2 by -4

pg. 34 Prepared by: Firehun A


[ 1 3 2 30
|
0 1 3/4 6 ]
and add -3 times row 2 to row 1:

[ 1 0 −1/4 12
|
0 1 3/4 6 ]
At this point, the method terminates since there is no entry in the third row and third column. In
fact, there is no third row at all. Thus we have an unlimited number of solutions given by
1
x +0− z=12
4
3
0+ y+ z=6
4
or solving both equations in terms of z,
1
x= z +12
4
3
y=− z +6
4
In this situation, we say z is “free” and can take on any real number as a value. Once a value is
chosen for z, the corresponding value for x and y can be found from the above equation.

Gauss Jordan Elimination Method- Steps


 Choose the leftmost nonzero column and use appropriate row operation to get a 1 at
the top.
 Use multiples of the first row to set zeros in all places below the 1 obtained in step 1.
 Delete (mentally) the top row and first column of the matrix. Repeat step 1 and step 2
with the sub matrix. Continue the process considered in steps of 1 – 3, until it is not
possible to go further.
 Consider the whole matrix obtained after mentally returning all the rows and columns
to the matrix. Begin with the bottom nonzero row and use appropriate multiples of it
to get zeros above the leftmost 1. Continue this process, moving up row by row until
the matrix is finally in a reduced from

Example: Solve the following systems of linear equation by Gauss Jordan method.

pg. 35 Prepared by: Firehun A


2x – y = 4
-6x + 3y = -12
To solve the above linear equations by Gauss Jordan method
First, transform the given system into an equation that involve matrices, namely, the coefficient
matrix, the variable matrix, and the constant matrix as follows:

[−62 −13 ][ xy ]=[−124 ]


Then form an augmented matrix using the coefficient matrix and the constant matrix and then
apply the row operations, as show below;

[ 2 −1 4
|
−6 3 −12 ] 1
2
R1 → R1 [
1 −1/2 2
|
−6 3 −12 ]
6 R1 + R 2 → R2 [10 −1/20 |20 ]
By referring to number 2, above the system has infinitely many solutions, that is the general
solution of the system is given below
x -½y=2

x = ½ y + 2 if y = c, where c is any real number, then x = ½ c + 2.

The general solution set is {(½ c + 2, c)}, which is infinitely many.


Example: Solve the following system linear equation using the Gauss Jordan elimination
method.
3x – 4y = 1
5x + 2y = 1
To solve the above linear equations by Gauss Jordan method
First, transform the given system into an equation that involve matrices, namely, the coefficient
matrix, the variable matrix, and the constant matrix as follows:

[ 35 −42 ][ xy ]=[ 11 ]
then form an augmented matrix using the coefficient matrix and the constant matrix and then
apply the row operations, as show below;

pg. 36 Prepared by: Firehun A


[ 3 −4 1
|
5 2 1 ] 1
3
R1 → R 1 [
1 −4/3 1/3
5 2 1
|
]
−5 R1 + R 2 → R2 [10 −4/3 1/3
|
26 /3 −2/3 ]
3
26
R 2 → R2 [
1 −4/3 1/3
|
0 1 −1/13 ]
4
R + R 1 → R1
3 2 [0 1 −1/13]
1 0 | 3/13

Hence, x = 3/13 and y = -1/13


There fore, the solution set is {(3/13, -1/13)}. Note that this linear system of equation has a
unique solution.
Example: Solve the following system of linear equations using the Gauss Jordan elimination
method.
x + 3y = 2

2 x + 6y = -3
First, transform the given system into an equation that involve matrices, namely, the coefficient
matrix, the variable matrix, and the constant matrix as follows:

[ ][ ] [ ]
1 3 x
=
2 6 y −3
2

then form an augmented matrix using the coefficient matrix and the constant matrix and then
apply the row operations, as show below;

[ 1 3 2
|
2 6 −3 ] −2 R1 + R 2 → R2 [ 1 3 2
|
0 0 −7 ]
Hence, x + 3y = 2
0x + 0y = -7
This implies 0 = 7 that is false. Therefore, the solution set of this system is Ø. That is, the system
has no solution.
Example: A garment factory produces three types of shirts, small, medium and large models.
Each requires the following services of the three departments as listed in the table. The cutting,

pg. 37 Prepared by: Firehun A


tailoring and packaging departments have available a maximum of 380, 300 and 120 labor –
hours per week respectively. How many shirts of each type must be produced each week for the
factory to operate at full capacity?

Type of Shirts Available


Department
Small Medium Large Labor Hours
Cutting 0.5 1 1.5 380
Tailoring 0.6 0.9 1.2 330
Packaging 0.2 0.3 0.5 120

To solve the above application problem you have to follow the following steps. First let us fit the
question with a mathematical model that involves linear equations on the basis of the given
assumptions.
Let as assume the total number of different sized shirts as x, y and z.
x = the number of small size shirts produced per week.

y = the number of medium size shirts produced by the factory per week.
z = the number of large size shirts produced by the factory per week.

Therefore,
The total labor-hours in the cutting department is
0.5x + y +1.5z = 380
The total labor-hours in the assembling department is
0.6x + 0.9y + 1.2z = 330.
The total labor-hours in the packaging department is
0.2x + 0.3y + 0.5z = 120.
Therefore, the linear system model of the problem is obtained to be
0.5x + y + 1.5z = 380.
0.6x +0.9y + 1.2z = 330.
0.2x +0.3y + 0.5z = 120

pg. 38 Prepared by: Firehun A


Find the three matrices, i.e. the coefficient matrix (A), the variable matrix (X) and the constant
matrix (C), and form the matrix way of representation of the system.

[ ] [] [ ]
0.5 1 1.5 x 380
0.6 0.9 1.2 y 330
A = 0.2 0.3 0.5 , X = z and C = 120
Thus, involving the above matrices one can easily represent the above system as

[ ][ ] [ ]
0.5 1 1.5 x 380
0.6 0.9 1.2 y 330
0.2 0.3 0.5 z = 120
Now let us form an augmented matrix using the coefficient matrix and the constant matrix and
then apply the row operations, as show below;

[ ] [ ]
0.5 1 1.5 380 1 2 3 760
0.6 0.9 1.2 |330 0.6 0.9 1.2 |330
0.2 0.3 0.5 120 2 R1 → R1 0.2 0.3 0.5 120

[ ]
1 2 3 760
−0 . 6 R1 + R2 →R 2 0 −0.3 −0.6 |−126
−0 . 2 R 1 + R3 →R 3 0 −0.1 −0.1 −32

[ ]
1 2 3 760
1 0 1 2 |420
− R 2 → R2 0 −0.1 −0.1 −32
0 .3

[ ]
1 0 −1 −80
0 . 1 R2 + R3 →R 3 0 1 2 | 420
−2 R2 + R 1 → R1 0 0 0.1 10

[ ]
1 0 −1 −80
0 1 2 |420
10 R3 →R 3 0 0 1 100

[ ]
1 0 0 20
−2 R3 + R 2 → R2 0 1 0 |220
1 R 3 + R1 →R 1 0 0 1 100
Hence, x = 20, y = 220 and z = 100.

pg. 39 Prepared by: Firehun A


Therefore, 20 small size shirts, 220 medium size shirts and 100 large size shirts can be produced
per week at full operation capacity of the garment factory.

2.5.1.2 Inverse method


Here you are going to apply the inverse of a given square matrix, if one exists, in finding a
solution of n by n systems of linear equations. After you have learned computational procedures,
you will discover that they are directly applicable to get the solution of n by n systems of linear
equations. Before studying this Section, please revise identity matrices, inverse of square
matrices, multiplication of matrices and the elementary row operations.
Consider the statement
A = IA,
Where A is an n by n square matrix and I is the corresponding identity matrix.
Suppose that we carry out row operations on the left of the equal sign to change the left matrix to
I, and maintain the equality by applying the same operations to the first matrix on the right of the
equal sign [which starts out as I]. The end result will be I on the left and something times A on
the right; thus, we have expression of the form
I = [?] A.
I showed earlier that the inverse of a matrix is unique. Hence trhere is only one appropriate entry
−1
for [?] in the above equation. It is A . It follows that if we write a given square matrix A with I
to its right, then change the left to I by row operations that are applied also to the right, the end
−1
result will be A on the right.
Symbolically, then, the Gauss Jordan method for finding the inverse of a square matrix
[ A|I ]
and use the elementary row operations to transform the latter matrix into

[ I|A−1] .

pg. 40 Prepared by: Firehun A


Of course, the above process will work only when there exists an inverse. Now clearly not every
square matrix has an inverse. For example, the zero matrix, O, has no inverse since O⋅B=O for
every matrix B.

Now I will show you how the solution of an n by n system of linear equations is accomplished
by the inverse of a square matrix. To set the stage, consider first the manner in which a simple
linear equation such as
2x = 3
is solved for x. To bring out what we have in mind, tow procedures using slightly different
symbols are presented side by side.
Regular solution symbol Inverse solution symbol
2x = 3 2x = 3

( 12 )(2 x )=( 12 )(3) ( 2−1 ) (2 x )=( 2−1 ) (3


1
( 1 ) x= (3 )
2 ( 1 ) x=( 2−1 ) (3)
1
x= (3 )
2 x=( 2−1 ) (3 )
Notice that on the right that the solution was obtained by multiplying both sides of the original
−1
equation by the multiplicative inverse of the coefficient of x, 2, which is2 .
The procedure of a linear system proceeds in a manner analogous to that illustrated for a single
equation, except that now we multiply both sides of the matrix form of the system by the inverse
of the coefficient matrix. To see how this works out, consider the 2 by 2 system
2 x+3 y =17
x +2 y=10
The system can be written in expanded matrix form as

[ ][ ] [ ]
2 3 x 17
=
1 2 y 10
or equivalently in the form
Ax=b
In this equation,

pg. 41 Prepared by: Firehun A


2 3
A= 1 2
[ ]
is the matrix of coefficients,

x=
[ xy ]
is the column matrix of variables, and

b= [ 1710 ]
is the column vector of the constants. If both sides of the matrix equation Ax=b are multiplied
−1
by the inverse of the coefficient matrix, namely, A , we have
A−1 Ax= A−1 b
from which it follows that,
x= A−1 b
−1
Now recall that we solved for the inverse matrix x= A b by using row operations to transform
the matrix
[ A|I ]
into the matrix

[ I|A−1] .
If we then start with the augmented matrix
[ A|b ] ,
we can rewrite this in the form

[ A| I⋅b ]
Using the row operations, we would be transforming the latter matrix into

[ I| A−1⋅b ]
Thereby giving the solution to the system of equations.
Example: Use inverse of the coefficient matrix to solve the following system of linear equations.
2x – 3y = 4
x + 5y = 2

pg. 42 Prepared by: Firehun A


To solve the above system of linear equations carry out the following general procedures:
Find the coefficient matrix (A), the constant matrix (b) and the variable matrix (x). That is,

A=
[ 2 −3
] []
4 x
1 5 , b = 2 and x = y []
−1
Find A , by using the three elementary row operations on the augmented matrix, and it may be
verified that the inverse from the augmented matrix

[ 1 0 5/13 3/13
|
0 1 −1/13 2/13 ] as

A−1 =
[ 5 /13 3/13
−1 /13 2/13 ]
Hence the solution vector is

[ xy ]=[−1/13
5/13
=
][ ] [ ]
3 /13 4 2
2 /13 2 0
Hence, x = 2 and y = 0.
Therefore, the solution set is {(2,0)}.
Example: Suppose a company makes liquid products Prime oil, Mid oil, and Last oil, which

contain different amounts of additives A1 , A2 , and A3 per gallon, as shown below.

Pounds of Additive per Gallon


Liquid Gallons Made

A1 A2 A3

Prime oil x1 1 1 1
Mid oil x2 1 2 1
Last oil x3 2 0 1

The additive deteriorate if not used within a week, so each Saturday the company schedules

production of [
x1 x2 x3 ]
gallons of Prime oil, Mid oil, and Last oil to use up the additives on

hand. These amounts vary form week to week and are represented by the vector [
a1 a2 a3 ]
. If

pg. 43 Prepared by: Firehun A


we schedule [ x1 x2 x3 ]
gallons of the liquids, the pounds of additive A1 used will be
x 1 (1 )+ x 2 (1)+ x 3 ( 2 ) , and this should equal a 1 , the amount available. Hence,
x 1 + x 2 +2 x 3 =a1 . Combining this condition with the conditions on additives A2 and A3 leads

us to the system
x 1 + x 2 +2 x 3 =a1
x 1+ x 2 =a 2
x 1 + x 2 + x 3 =a1

The coefficient matrix is

[ ]
1 1 2
A= 1 2 0
1 1 1
and it may be verified that the inverse is

[ ]
−2 −1 4
−1
A = 1 1 −2
1 0 −1
Hence the solution vector is

[][ ][ ]
x1 −2 −1 4 a1
x2 = 1 1 −2 a2
x3 1 0 −1 a
3

Suppose that on a given Saturday the amounts of additives available are a 1 =20 pounds of A1 ,
a 2 = 30 pounds of A2 , and a 3 = 20 ponds of A3 . To use up these additives, the production

schedule should be

[][ ][ ] [ ]
x1 −2 −1 4 20 10
x2 = 1 1 −2 30 = 10
x3 1 0 −1 20 0

which is 10 gallons of Prime oil, 10 gallons of Mid oil, and no Last oil.
Example: If 3 Kilos of sugar and 2 kilos of coffee cost Br. 35 while 5 kilos of sugar and 3 kilos
of coffee cost Br. 55, find the price of sugar and coffee per kilo.

pg. 44 Prepared by: Firehun A


Let 1 kilo of sugar costs = Br. x and 1 kilo of coffee costs = Br. y.
3x + 2y = 35
5x + 3y = 55
Find the coefficient matrix (A), the constant matrix (b) and the variable matrix (x). That is,

A=
[ 35 23 ] [ 3555 ]
,b= and x =
[ xy ]
−1
Find A , by using the three elementary row operations on the augmented matrix, and it may be
verified that the inverse from the augmented matrix

[10 0 −3 2
|
1 5 −3 ] as

A−1 =
[−3 2
5 −3 ]
Hence the solution vector is

[][
x −3 2 35
y
=
5 −3 55
=
5
10 ][ ] [ ]
Hence, x = 5 and y = 10.
Thus, the cost of a kilo of sugar is Br. 5 and the cost of a kilo of coffee is Br. 10.

2.5.2: Markov Chain: Concepts, Model and Solution


In this sub-section you will study the application of matrix and probability. Since the sub-section
is dependent on probability, equality of matrices and solution of systems of linear equations,
please try to make a review of the above stated concepts before you move to this section of the
unit.

Definitions:
1. Markov Chain is a sequence of experiments (trials) such that the probability for the
next states is completely determined by the present state.
2. Transition matrix is a square matrix having no negative entries and the sum of entries
in each row is equal to 1.
3. Probability matrix is a row matrix having no negative entries and the sum of the
entries is equal to 1. A probability matrix that gives a long-range prediction for a
pg. 45given transition matrix is called fixed probability matrix. That is, if PPrepared
is a transition
by: Firehun A
matrix, then the probability matrix V is said to be a fixed probability matrix if and
To set the stage for this section let us consider the following example.

Example: Suppose that a market chain notes 30 percent of the dinners it sells each week are
'Dorowott' and the remaining 70 percent are other dinners. The chain manager has a special
arrangement for volume buying of 'Dorowott' at a relatively low price, and would like to raise the
promotion of ‘Dorowott’ dinners sold.
After the promotional campaign has been carried out it was observed the following transition
proportions.
Table 1: Transition proportions, One Week to Next Week
One Week Next Week
'Dorowott' Other
'Dorowott' 0.8 0.2
Other 0.6 0.4

Note that:

1. The matrix proportion


[0.80.6 0.20.4 ] is called Transition Matrix, and
2. The state of affairs at the beginning of the section was 30 percent 'Dorowott' and 70

percent other. We shall call the row vector ( 0.3 0.7 ) the Initial State Vector.

If we wish to find what proportion will buy 'Dorowott' after one week transition we note that 80
percent of the 30 percent who bought 'Dorowott' one week buy it next, and an additional 60
percent of the 70 percent who bought Other dinners one week will buy 'Dorowott' next week.
Thus, the proportion buying 'Dorowott' after one week transition is obtained to be

pg. 46 Prepared by: Firehun A


0 . 3⋅( 0 . 8 ) +0 .7⋅( 0 . 6 )=0. 66 . Similarly, the proportion of buying other dinners after one week

transition is obtained as 0 . 3⋅( 0 . 2 )+ 0 .7⋅( 0 . 4 ) =0 .34


We find this using the usual inner product principle of matrices as shown below:

[ ]
( 0.3 0 .7 )⋅ 0.8 0.2 =( 0 .66 0 .34 )
0.6 0.4
Now suppose that the promotion activity is maintained and the transition matrix remains constant

from week to week. Therefore, the state vector for week 1, ( 0. 66 0. 34 ) , can be used as a pre-
multiplier of the transition matrix to obtain the state vector for week 2.
Table 2: Successive State Vectors
Week Number 'Dorowott' Other
Beginning (0.30 0.70)
1 (0.66 0.34)
2 (0.732 0.268)
3 (0.7464 0.2536)
4 (0.74928 0.25072)
5 (0.749856 0.250144)
. . .
. . .
. . .

Note that:
1. The component of each state vector is added up to a 1.

2. The state vector appears to be approaching (0.75 0.25)

What would happen if we use the state vector(0.75 0.25) as a pre-multiplier of the transition
matrix to obtain the state vector for the succeeding week;

(0.75 [
0.25) 0.6 0.4 ] (0.75
0.8 0.2
= 0.25)

pg. 47 Prepared by: Firehun A


It leads to the same state and we call this state the Steady State.
Let us see how to find the Steady State at the initial moment of the market chain by method other
than tabulating the successive state vectors and guessing the Steady State from the sequence of
the results.

Let us assume that the Steady State be a vector( x y ).

Hence, the matrix multiplication 0.6 0.4 [ ]


( x y )⋅ 0 . 8 0. 2 =( x y )
and the equation x+y=1
yields x = 0.75 and y = 0.2
Thus, it is possible to note that the promotional campaign will be seized when the proportions of
'Dorowott' users reached 75 percent because it is not possible to exceed more than this no mater
the promotion will be sustained longer.

Next let us suppose that once a customer buys ‘Dorowott’ he or she is not satisfied that he or she
will buy ‘Dorowott’ the next time. This results in a 1 in the upper left corner of the transition
matrix. Thus,

[ ]
Dorowott Other
Borowott Other
( 0. 3 Dorowott 0. 8 0 .2
0 .7 ) ⋅
Other 0. 6 0.4
In as much as 60% of those buying other meals change to ‘Dorowott’, then continue to buy
‘Dorowott’, it reasonable to expect that ultimately all customers will buy ‘Dorowott’ and the
steady state will be (1 0). In this situation, ‘Dorowott’ has absorbed all the business, and the
chain leading to this steady state is called an absorbing chain.
2.5.3 Input-output Model
Here you study one of the very important applications of matrices and inverses of matrices found
in the relatively recently developed branch of applied mathematics called input-output analysis.
Input-output analysis is an application of matrices and their inverses, and attempts to establish
equilibrium conditions under which industries will have just enough output to satisfy each
other’s demands in addition to outside demands.
In this section I will give much emphasis in determining output levels for the various industries
that will meet a given outside level of demand as well as the internal demand given the internal
demands for each industry's output

pg. 48 Prepared by: Firehun A


The technology matrix is the heart of input-output analysis. The elements in the technology
matrix are determined as shown below.

Given two companies c 1 and c 2 whose technology matrix, output matrix and final demand
matrix are described as follows:

Technology matrix Output matrix outside demand matrix


c1 c2

A=
[
c 1 a 11
c 2 a 21
a12
a22 ] X=
[ ]
x1
x2
D=
[ ]
d1
d2

Where
a ij is the input required from c i to produce a Br’s worth of output for c j . Now let us
form the solution to the input-output equation. The solution to the input-output matrix equation
is:
Total output = Internal demand + Outside demand.
This might have been explained using the above three matrices as:
X = AX + D
Solving for the variable X leads us to have,
X – AX = D
(I – A)X = D

( I− A )−1 ( I− A ) X=( I− A )−1 D , assuming that I – A has an inverse.

X =( I− A )−1 D

where I = the identity matrix


A = the technology matrix
D = the final demand matrix
X = the output matrix.
Example: An economy is based on two industrial sectors, rubber and leather. The rubber
producing company uses both rubber and leather (input) in the production of rubber (output) and
leather producing company uses both rubber and leather (input) in the production of leather

pg. 49 Prepared by: Firehun A


(output). Suppose that the production of each Br’s worth of rubber requires Br. 0.10 worth of
rubber and Br. 0.30 worth of leather, and the production of each Br’s worth of leather requires
Br. 0.40 worth of rubber and Br. 0.20 worth of leather.
a. Find the total production necessary to produce for a final consumer demand of Br. 12,
000,000 of rubber and Br. 6,000,000 of leather.
b. Suppose the project final or outside demands 3 years from now were Br. 18 million of rubber
and Br. 12 million of leather; determine each total output for the project.
a. In order to solve these problems, you are required to construct the technology matrix, A, the
−1
output matrix, X and outside demand matrix, D, the inverse matrix ( I− A ) . And then find

X =( I− A )−1 D , which is total output required to satisfy the final demand.

Let x 1= Total output of rubber, and x 2 = Total output of leather be assumed.


Technology matrix Output matrix Outside demand matrix
Rubber Leather

[
Rubber 0 .10 0 . 40
A = Leather 0 .30 0 .20
] X=
[ ]
x1
x2
D= 6
[ ]
12

Then,

I–A=
[ 10 01 ]− [0.10
0.30
0. 40
][ 0.90 −0. 40
0.20 = −0 .30 0 .80 ]
−1
Then using the Gauss-Jordan Inversion technique the inverse of matrix I – A, ( I− A ) is
obtained to be:

−1 [
4 /3 2/3
( I− A ) = 1/2 3/2 . ]

Therefore, the total output required to satisfy the final demand,

−1
X =( I− A ) D = 1/2 3/2[
4 /3 2/3
][ ] [ ]
12 20
6 = 15
Hence, Br. 20 million of rubber and Br. 15 million of leather is required to satisfy a final
demand of Br. 12 million of rubber and Br. 6 million of leather.
b. It is left as an exercise for you.

pg. 50 Prepared by: Firehun A


___________________________________________________________________________
___________________________________________________________________________
_________________________________________________________

Since D =
[ 1812 ] , you are expected to obtain Br. 32 million of rubber and Br. 27 million of
leather is required to satisfy a final demand of Br. 18 million of rubber and Br. 12 million of
leather.

pg. 51 Prepared by: Firehun A


UNIT 3
Introduction to Linear Programming Problems
3.1 Basic Concept of Linear Programming
In this Section you will learn the fundamental concepts of linear programming problems. I will
also discuss and introduce to you various types’ terminologies, which are often raised in the
construction and finding solutions of linear programming problems.
Linear programming is a mathematical technique, which aids managers in making the best use
of a firm's economic resources. Note that the variables in real-life situations are subject to
restrictions that we shall call constraints. Let us suppose that the relevant variables in a certain
situation are x and y, and that the constraints can be expressed as a system of inequalities in these
variables. Then suppose we have an expression called the objective function, which states how
profit (or cost) is computed for specific value of x and y. The problem is to find the set of values
for x and y that satisfies the constraints and maximizes profit (or minimizes cost). If the
constraints and the objective functions are linear, we have a problem in linear programming.

A typical problem of this type is found in the case of a manufacturer who produces four different
products each of which requires a specified amount of time in each of five processes. In each of
the five processes only a limited number of hours are available per time period. Furthermore the
profit per unit of output is different for each of the five products.

The problem that must be solved requires the optimization of output with the available resources.
The optimal output in this context in this context is the number of units of each product that
should be produced to maximize the total profit.
To be more specific, suppose that a textile mill buys unfinished cloth and uses 10 processes to
convert in to 12 styles of finished material. The styles require varying amounts of the item
available for each process, and there is a capacity limitation on the time available for each

pg. 52 Prepared by: Firehun A


process, making 10 capacity constraints. Further, a certain quantity (at least) of each style must
be produced to satisfy customer demand, making 12 demand constraints, so we have a total of 10
+ 12 linear inequality constraints, not counting nonnegative constraints.
Note that simple linear programming problem involving two products only can be solved using
two-dimensional graphs; multi product problems require the use of more advanced algebraic
techniques.

To deal with the two-product case graphically the following approach should be taken.

1. State the problem in algebraic terms.

a. Represent the number of units of each product by x 1 and x 2 respectively and organize
the data in the form of a chart.
b. State the Objective Function in equation form. The Objective function is a mathematical
presentation of the goal to be achieved-either as a profit, which is to be maximized, or as
a cost which is to be minimized.
c. List the Operational Constraints imposed on the objective function. The operational
constraints indicate that the total amount of each type of economic resource used has to
be consistent with the available amount of each resource.

d. List the Non-negative Constraints. These constraints indicate that x 1 and x 2 cannot be
less than zero; that is , the number of units produced cannot be negative.
2. Graphically represent the algebraic relationships.
a. Represent the constraints graphically in a two-dimensional system or rectangular
coordinate system to obtain the area feasibility.
b. Introduce graphs of the objective function to identify the optimal point. The optimal

point is the point ( x 1 , x 2 ) for which the profit will be maximized or the cost will be
minimized. If a single solution exists, the optimal point is always point on the boundary
of the area of feasibility.
3. Algebraically determine the coordinates of the optimal point to find the optimal solution and
the value of the objective function at that point.
Generally, linear programming problem is one that is concerned with finding the maximum or

minimum value of a linear objective function of the form Z=c 1 x 1 +c 2 x 2 +c 3 x 3 + ¿ ⋅⋅+c n x n ,

pg. 53 Prepared by: Firehun A


Where the decision variables are, x 1 , x 2 , x 3 , ⋅⋅⋅, x n , subject to problem constraints in the
form of linear inequalities and equations. In addition the decision variables must satisfy the non-
negative constraints
x i > 0,

where i = 1, 2 , 3, . . .n.
The region bounded by the operational constraints and non-negative constraints is called
feasible region.
If a linear programming problem has an optimal solution, then this solution must occur at one or
more of the corner points of the feasible region.

3.2 Techniques of Solving Linear Programming Problem


In this section, you will learn about determining the solutions of linear programming problems
by graphical method involving maximization and minimization problems of product mix.
This Section deals mainly on graphs of linear equations and linear inequalities. Therefore, the
Section requires you to review the concept of graphs of linear equations and graphs of linear
inequalities. In this section, you will learn about the feasible regions, the corner points and
formation of the objective functions together with the constraints which hinder their
determination. Keep in mind that the following points will be attained after the discussion in
this section
 draw the graph of linear programming problems;
 identify the feasible regions of linear programming problems;
 locate the corner points of the feasible region; and
 tell the optimal solution of the linear programming problems.

3.2.1 Graphical Method of Solving Linear Programming Problem


To solve linear programming problems by graphical methods you can follow the steps listed
below:
a. Form a mathematical model for the linear programming problems, that is;

i. introduces the decision variables x 1 , x 2 , x 3 , ⋅⋅⋅, x n and write a linear

pg. 54 Prepared by: Firehun A


Objective function, Z=c 1 x 1 +c 2 x 2 +c 3 x 3 + ¿ ⋅⋅+c n x n ; and
ii. Write problem of constraints using linear inequalities and/or equations.
b. Graph the operational constraints function and find the feasible region and the corner
points.
c. If an optional solution exists, evaluate the objective function at each corner point to
determine the optimal solution.
To have a thorough understanding of the above steps and to obtain the solution of linear
programming problems, let us consider the following example.

Example: A manufacturer markets two products. Each unit of Product A requires 3 hours in the
molding department, 4 hours in the paint shop and 1 hour in finishing. Each unit of product B
requires 3 hours in molding department, 2 hours in the paint shop and 2 hours in finishing. Each
week there are 210 hours available in molding, 200 hours in painting and 120 hours in finishing.
Shipping can handle no more than 40 units of product A per week. Each unit of product A
contributes Br. 20 to profit while each unit of product B contributes Br. 30. Determine how many
units of each product should be manufactured per week to maximize profit.
Step 1: Algebraic statement of the linear programming problem
a. Let x be the number of units of product A and y be the number of units of product B.

Data Summary
Resource quantity per unit
Constraints Available resource quantity
for
Product A Product B

Molding 3 3 210
Painting 4 2 200
Finishing 1 2 120
Shipping 1 - 40
Profit Br. 20 Br. 30 Maximize

b. The Objective Function:

pg. 55 Prepared by: Firehun A


The goal is to maximize profit. p may represent the value of the total profit. The amount of
profit contributed by product A is Br. 20 per unit or Br. 20x and the profit contributed by
product B is Br. 30 per unit or 30y. The total profit is p = 20x + 30y, the objective
function then is p = 20x + 30y with x and y to be chosen so that p becomes as large as
possible.
c. The Optional Constraints:
Production in the given situation depends on limited depends on limited resources; the
availability of time in the molding department, in the paint shop, and in finishing as well as
the ability of shipping to handle product A.
Each of the constraints on the operations of the business can be expressed algebraically by
relating the resource requirements per unit of product to the total resource available.
For the molding department each unit of product A requires 3 hours, as does each unit of
product B. The total time used in the molding department to manufacture the two products
must be less than or at most equal to 210 hours per week;
That is,
3x + 3y ¿ 210.
Similarly with respect to the paint shop,
4x + 2y ¿ 200, and
with respect to finishing
x + 2y ¿ 120.

As far as shipping is concerned, the total number of units of Product A must be less than or
equal to 40; That is , x ¿ 40.
d. The non-negative constraints.
The minimum number of units of each product that can be produced is zero. This fact is
represented by the inequalities x ¿ 0 and y ¿ 0, referred to as the non-negative constraints.
Step 2: Graph the operational constraint and the non-negative constraints inequalities on the
same plane.
Note that, each constraint involves an inequality, which, when graphed, represents a region
referred to as the area of feasibility for the resource represented by the inequality. First graphing
the associated equality and testing for the region can graph each inequality.

pg. 56 Prepared by: Firehun A


a. Graphing the area of feasibility.
The area of feasibility for the combined resources is the area containing the points, which
satisfy all operational constraints. This area is found by graphing the individual constraints
on the same set of axes, shown below.
Area of Feasibility

b. Locating the Optimal Point


Once the area of feasibility has been determined by graphing the operational constraints, the
operational point, the point whose coordinates are the combination (x, y), which generates the
maximum profit-needs to be located. The feasible region is the region which is common to
all the constraint functions. And the optimal point is a point in the boundary of the area of
feasibility. Note that, this is generally true for any linear programming problem.

pg. 57 Prepared by: Firehun A


Step 3: Evaluate the objective function at each corner points to determine the required optimal
solution.
The corner points can be obtained by the ordinary solutions of system of linear equations.
Vertices or the feasible Values of
region (corner points) p = 20x + 30y
(0, 0) Br. 0.00
(40,0) Br. 800.00
(40,20) Br. 1400.00
(30,40) Br. 1800.00
(20,50) Br. 1900.00
(0,60) Br. 1800.00

20 units of product A and 50 units of product B must be manufactured to maximize profit and the
maximum profit is Br. 1900.
Example: First let us introduce the decision or predictor variables:
Let x and y be the total number of lounge chairs and swivel chairs made in the company
respectively.
And now let us fit a mathematical model that will explain the objective function and the
constraints observed in the company.
The profit achieved or the Objective Function p from these chairs can simply be stated as
follows:
p = [Profit per lounge Chair]  [Number of Lounge Chairs made and sold] + [Profit per
Swivel Chair]  [Number of Swivel Chairs made and sold]
Symbolically,
p( x , y )=1x+0 .5 y
To make x lounge chairs and y swivel chairs Department A will requires x + y hours, and
Department B will requires x + 2y hours.

pg. 58 Prepared by: Firehun A


The 4 hours’ time limitations of Department A and the 6 hours’ time limitation of Department B
leads us to have inequalities called Problem Constraints.
x + y≤4 and
x +2 y≤6 .
The above constraints taken together with Non-negative Constraints on x and y, namely x≥0

and y≥0 brings to the attention of the following statement of the problem.
Maximize the objective function p=x+0.5 y , which is subject to the constraints
x + y≤4
x +2 y≤6
x≥0
y≥0
The Solution Space or the Feasible Region of the above system will be shown as follows:
Area of Feasibility

Note:
1. The Corner Points of the feasible region are labeled by A, B, C, and D.
2. Each point in the region will have a corresponding order pairs of real numbers of the form (x,
y), where x and y are the number of lounge chairs and swivel chairs made in the company
respectively.

pg. 59 Prepared by: Firehun A


Now by evaluating the profit for all points in the feasible region, specifically on the corner points
it is possible to observe that maximum profit can be obtained when 4 lounge chairs and 0 swivel
chairs are made and sold.

Example: A farmer raises only cows and sheep. He wants to raise no more than 16 animals,
including no more than 12 sheep. He spends $5 to raise cows and $2 to raise a sheep. He has $50
available for this purpose. Cows sell for $10 and sheep for $5. How many of each should he raise
for maximum profit? What is the maximum profit?
Let x be the number of cows and y be the number of sheep that the person would raise.
Therefore,
a. Objective function: p = (10 – 5)x + (5 – 2)y = 5x + 3x
b. Operational constraints: 5x + 2y ¿ 50
x + y ¿ 16
y ¿ 12
c. Non-negative constrains: x ¿ 0, y ¿ 0
Now let us draw the graphs of the constraint functions and the non-negative constraints on the
same rectangular coordinate plane and find the feasible region and the corner points that bounded
the feasible region and evaluate the objective function at the corner points of the feasible region.

pg. 60 Prepared by: Firehun A


Vertices or the feasible Values of
region (corner points) p = 5x + 3y
(0, 0) $0.00
(0, 12) $36.00
(4, 12) $56.00
(6, 10 $60.00
(10, 0) $50.00

TH farmer must raise 6 cows and 10 sheep to obtain the maximum profit. The maximum profit is
$60.00.
Example: Minimize and maximize z = 3x + y subject to constraints
2x + y ¿ 20
10x + y ¿ 36

pg. 61 Prepared by: Firehun A


2x + 5y ¿ 36
x ¿ 0, y ¿ 0
To solve this linear programming problem, you need to draw the graphs of 2x + y ¿ 20, 10x
+ y ¿ 36, 2x + 5y ¿ 36, x ¿ 0, and y ¿ 0 as parts a plane and look for their intersection, which is
the right feasible region and then determine the corner points. Then evaluate the objective
function at each corner point to find the minimum and maximum value of the linear
programming problem.

Evaluate the objective function at the corner points of the feasible region.
Vertices or the feasible Values of
region (corner points) z = 3x + y
(2, 16) 22
(3, 6) 15
(8, 4) 28

z has a maximum value 28 at the point (8,4) and its minimum value is 15 at the point (3,6).

The following example is a bite different from the examples given above. This example is only
on minimization of the objective function, so the graph opens up ward as you will notice in the
solution below, you can find the situation in the solution below, but first try it and then go
through the solution.

pg. 62 Prepared by: Firehun A


Example: Minimize z = 10x + 20y Subject to the constraints
6x + 2y ¿ 36
2x + 4y ¿ 32
y ¿ 20 and
x ¿ 0, y ¿ 0
To solve the linear programming problem, graph the operational constraints on an x-y plane, and
find the feasible region, the corner points and evaluate z (the objective function) at each corner
point.
To graph the operational constraints, change the inequalities to equalities.
That is,
6x + 2y = 36, 2x + 4y = 32 and y = 20, and draw the graph of these linear equations. By selecting
testing point determine the part of the plane, which is considered as a graph of the corresponding
inequality (constraint). Next find the feasible region (intersection of the graph of the inequalities)
and the corner point, and evaluate the objective function, z, at each corner point to determine the
optimal solution.

Evaluate the objective function at the corner points of the feasible region.

Vertices or the feasible Values of

pg. 63 Prepared by: Firehun A


region (corner points) z = 10x + 20y
(0, 18) 360
(4, 6) 160
(16, 0) 160
(0, 20) 400
z has minimum value at (4,6) and (16,0) and the minimum value is 160. This is a multiple

optimal solution.

Note:
If two points are both optimal solutions to a linear programming
problem, then any point on the line segment joining them is also an
optimal solution.

UNIT 4
Mathematics of Finance
4.1 Over view of Exponential and Logarithmic Functions
The objectives of this section is to review the nature and properties of exponents , exponential
functions logarithms and logarithmic functions. . We will make use of these functions to the
mathematics of finance.
4.1.1. Exponential functions
The general form of exponential function is given by : f ( x )=a x , a> 0
The general form of exponential function is given by : f ( x )=a x , a> 0
Where the value of a is fixed and the exponent x varies over all real number. Such a function is
called exponential function since the variable now appears as an exponent.

Properties of Exponents

For a , b> 0
 a 0=1
 a x . a y =a x+ y

pg. 64 Prepared by: Firehun A


x
a x− y
 y
=a
a
y
 ( a x ) =a xy
 ( ab )x =ax . b x

()
x x
a a
 = x
b b
−x 1
 a = x
a
In many applications of mathematics of finance, the natural exponential function is given by
x
f ( x )=e , where e ≈2.7182818 … … . is an irrational number like π.

Example: Make use of the above rules to evaluate the following

1. 2
e .e
−3

3
2
2. 4
2

3. ¿

Solution:

1. e 2 . e−3
2±3 −1 1
= e =e =
e
3 3
2 2 3−4 −1 1
2. 2 = 4 =2 =2 = 2
4 (2 )
3. ¿

4.2.2. The logarithmic Functions


In order to perform calculations like 100 ( 1.00560 ) , we need to define a function that is the
inverse of exponential function. On the other hand , logarithms are necessary in the solution of
certain type of exponential equations ,like 10 x =5.

Definition (of Logarithmic Function)

Let y=a x be an exponential function with a> 0. The logarithmic function

pg. 65 Prepared by: Firehun A


log a y =x if and only if y=a x. The domain of logarithmic function is the set of positive real
numbers and its range is the set of all real numbers. For example to solve 10 x =5 we need to
change it to log 10 5=x . Make use of log table to find the exact value.

Other examples like 23=8 means log 2 8=3

log 5 125=3 means 53=125 , log 2 ( 12 )=−1 means 2 =


−1 1
2

In general , log b M =x means b x =M

Common logarithm
The logarithm of a positive number to the base 10 , that is , log 10 a. It is denoted as log a is
called Common logarithm

Rules of Logarithms

 Product rule : log x ab=log x a+ log x b

 Quotient rule: log x


a
b ()
=log x a−log x b


n
Power rule : log x a =n . log x a and log x a ¿
(n )
( 1n ) log ax

Example : Solve the following

1. 2 x =8

()
x
1
2. =9
3
3. 27=9 x

Solution :

1. 2 x =8 means log 2 8=x , since 23 =8 , x=3

()
x
1 2
2. =9 means log ( 1 ) 9=x →−1. log 3 9=−2 since 3 =9
3 3

3 2
3. 27=9 x means log 9 27=x , log 3 3=
2 3

Natural Logarithmic function

The natural logarithmic function is log e x= y , denoted by Lnx= y , the logarithm of x to the base
e , which an irrational number as defined above .

pg. 66 Prepared by: Firehun A


4.2. Simple Interest and Discount
In this section, you will learn the basic concepts on simple interest and discount and Installment
Plan. Money transaction between lenders and borrowers usually requires compensation for their
services in the form of interest. The amount of such interest is calculated on the basis of three
factors: the amount of money borrowed the rate of interest at which the amount is borrowed, and
the time period for which it is borrowed.

4.2.1. Computing Simple Interest and Installment Plan


Simple Interest
When money is borrowed, a service charge is usually made for its use. The service charge is
called interest, and the sum of money invested or borrowed or lent is called present value
[principal]. The sum of the present value and the service charge on the present value for the time
it was in use is called future value [amount].
Interest rates are generally quoted in percentage form and, for use in calculations, must be
converted to the equivalent decimal value by dividing the percentage by 100; that is, by moving
the decimal point in the percentage two palaces to the left. For example,
1
i=8 %−8 . 25 %=0 . 0825
4
Unless otherwise stated, a quoted rate is a rate per year. Thus, $1 at 8% rate means that interest
of $0.08 will be earned in a year; and $100 at this rate provides
100(0.08) = $8
9
of interest in one year. Interest on $100 at 8 percent for 9 months is interest for 12 year; that is,

[ ]
9
Interest = 100 [0.08] 12 = $6.00
The last line introduces the following definitions, which apply in simple interest calculation.

Definition:
The simple interest

The amount of interest earned, I =Pin


Where I = the amount of interest earned;
P = Principal, the sum of money on which interest is being
pg. 67 Prepared by: Firehun A
earned;
i = rate of interest per period [assumed to be one year]; and
= number of years, or fraction of one year

Example: Find the interest rate if $1,000 earns $45 interest in 6 months.
6
Hear, I = 45, P = 1000, n = 12 = 0.5. Hence,
I =Pin
45=1000[ i][0 . 5 ]
45 = 500i
45
=0 . 09
i = 500

to obtain the percent rate, the decimal rate, i = 0.09, is multiplied by 100. Thus,
i = 9%
When time is given in days, there are two ways of computing the interest: the exact method and
the ordinary method [often called the Banker’s Rule]. If the exact method is used , then the time
is
Number of days
n=
365
but if the ordinary method is used, then
Number of days
n=
360
Banks, for convenience, often count a year as twelve 30-day months, 360 days for a year.
Example: Find the interest on $1,460 for 72 days at 10 percent interest using
a. the exact method
b. the ordinary method.
In both methods, P = 1460 and i = 0.1. For a.
I =Pin

I=(1460 )(0 .1 ) (72365 )


pg. 68 Prepared by: Firehun A
= $28.80
On the other hand, for b.
I =Pin

I=(1460 )(0 .1 ) ( )
72
360
= $29.20
As you can see from the results of the above example, the ordinary interest is more than the exact
interest.
The future Value: If interest on $1,000 at 9 percent for 8 months is computed as

I =Pin=(1000 )(0. 09 ) ( )8
12
=$ 60

And the interest is added to the principal, the sum is called the future value, F. Thus, the future
value is obtained to be

F=1000+(1000 )(0 . 09) (128 )=$ 1 , 060


Definition:
Future Value in the case of simple interest is defined by the formula
F=P+ Pin=P [1+in ].

Example: Find the total amount due on a loan of Br. 20,000 at 6 percent simple interest for 3
months.
3 1 1
= n=
Here 3 months is 12 4 of a year, so 4 . Hence,

[
F=20 , 000 1+0 . 06 ( 14 )]
=20 ,000 [ 1+0 . 015 ]
=Br. 20,300.
Example: Suppose after buying a new automobile you decide to sell your old automobile. You
accept a 6 month note for Br. 35,000 at 10 percent simple interest as payment. Sixty days later,

pg. 69 Prepared by: Firehun A


you find that you need the money and sell the note to a third party for Br. 35,500. What annual
interest rate will the third party receive for the investment?
First find the future value that will be paid at the end of 6 months to whomever has a note.

Hence,
[ ( )]
F=35 , 000 1+0 .1
1
2
=35 , 000(0 . 05 )=$ 36 ,750

And then for the third party we are to find the annual rate of interest required to make Br. 35,500
grow to Br. 36,750 in 4 months (6 months takeaway 60 days); that is, the rate will obtained given
4 1
n= =
future vale , F = $36,750, the principal P = $35,500, and 12 3 , as
F−P 36750−35500
i= = =0 . 1056=10 . 56 %
Pn
35500
1
3 []
Installment Plan
When you want to buy things you might find it difficult to pay the whole sum of money at the
time of purchase. So you may prefer to buy the thing now and pay later. Many business firms
offer installment’s plan to consumers who want to buy now and pay later. This scheme is also
known as extended payment plan. By installment plan you do not feel the pain of paying the cash
as you repay on easy monthly installments. It also gives us a feeling of possessing the article
before paying the whole cost. This plan has also its own disadvantages. The consumer do not
aware of the fact that the annual rate of interest he or she is paying is very high. Moreover, by the
time the buyer repays the whole amount, the article becomes worn out and outdated.
In installment purchase, the customer pays a certain amount or a percentage of the cash price on
the spot. The money thus paid at the time of purchase is called the Down Payment or Installment
Deposit. The remaining money still to be paid is called the Balance Due. Balance Due is also
name differently as Unpaid Balance or Outstanding Balance.

Definition:
Balance Due is defined as a difference between Cash Price and
Down Payment. Symbolically,
B D=C P −D P

pg. 70 Prepared by: Firehun A


The merchant charges the consumer a nominal for the privilege of using the article now and
paying later. The charge fee is called the Installment Charge. It is also known as Service Charge.
If a loan is to be paid in installments, then the interest is called the Loan Charge. In addition to
the down payment already made, the customer should pay the balance and the installment charge
by way of easy monthly payment. The amount that the customer repays every month is called
Monthly payments. Thus, the total cost of the article goes up by the installment plan. This total
cost is called the Installment Price, which is more than the cash price.
Note that:
Installment Price = Down Payment +[Monthly Payment ¿ number of months]

Symbolically, I P =D P + [ M P ×n ] , where n is number of months.


Installment Charge = Installment Price - Cash Price

Symbolically, I C =I P +C P , or
Installment Charge = [Monthly Payment ¿ number of months] – Balance Due
I C =[ M P ×n ]−B D
Symbolically,
The installment charge is considered as interest on the outstanding balance for the number of
months the installment is extended and the rate of interest thus calculated is called the Nominal
Rate.
100×Installment Charge
Nominal Rate [NR] = Unpaid Balance ×Time

Symbolically, NR=[ ( 100× I C ) / ( B D×T ) ] %


In addition to the down payment already made, the buyer has to pay the balance due and
installment charge. To make it more convenient for the buyer, the repayment of loan has been
distributed equally over the installment period. Hence,
Balance Due + Installment Charge
Monthly Payment = Number of installments
M P =( BD + I C ) /n
Symbolically,
The installment charge can be considered as interest on a loan. In other words after the down
payment, the balance due is treated as a loan borrowed from the seller to purchase the item.

pg. 71 Prepared by: Firehun A


100×Installment Charge
Effective Rate [ER] = Principal ×Time where time is in years.
NR×2 n
ER=
Or n+1 where n = number of installments.
Present Value: In as much as the future value, F, is obtained by
F=P(1+in)
we obtain the simple present value formula by dividing both sides by (1+in) . Thus,
F
=P
1+ in

Definition:
F
Present Value, P = 1+ in

Example: Find the present value of Br. 530 receivable 9 months from now if the interest rate is 8
percent.
530 530
P= =
1+0 . 08(9 /12 ) 1 .06 = Br. 500
4.2.2. Promissory notes and Bank Discount
In many loans, the interest charge is computed not on the amount the borrower receives but on
the amount that is repaid later. A charge for a loan computed in this manner is called bank
discount, and the amount the borrower receives is called the proceeds of the loan. Proceeds begin
with P and it is an amount received now. The future amount to be paid back is , now called the
maturity value of the loan . If Birr 1000 is borrowed at 12 % for 6 months , the borrower
receives the proceeds P , and pays back F=Birr 1000 . The proceeds will be Birr 1000 minus
the interest on Birr 1000. This will be
6
P=1000−1000(0.12) = 1000-60= Birr 940
12
The interest rate is denoted by d and is called bank discount rate . this method is also called
interest deducted –in –advance because the interest amount is deducted from the maturity value
F before the proceeds P are given to the borrower.

pg. 72 Prepared by: Firehun A


So the equation is given by p=F ( 1−dn ) , where n is time , usually months. In the above
illustration, the borrower receives Birr 940 but pays interest on the maturity value Birr 1000
Proceeds are an amount received now for a repayment in the future , so they are analogous to
present value that is discussed in the previous section .
A promissory note is a financial document that the borrower puts his signature.
Example: If a borrower receive proceeds of P=Birr 1000 at 12% interest for 6 months . What is
the maturity value ?
Using the above formula p=F ( 1−dn ), solving for F or substituting in the equation

(
1000=F 1−0.12
6
12 )
¿ F ( 1−0.06 )
= 0.94 F
1000
So F= = Birr 1063.83
0.94
Thus the borrower who wants Birr 1000 now will pay back Birr 1063.83

4.3. Compound Interest


In the previous session you have gone through the basic concept of interest which is usually
carried out for time period less than a year. In the previous lessons I have stated that if borrower
agrees to pay a fixed percent the amount borrowed in interest during each interest period, then
such interest is called simple interest. In this section, I will give emphasis on a more profitable
arrangement than simple interest, which is called a compound interest. In a compound interest
plan after each interest period the interest is added to the principal and interest is then received
on the sum of the principal and the interest and the interest accumulated in the previous interest
periods.

4.3.1. Maturity Value and Present Value


The term compounded interest refers to a procedure for compounding interest for a specified
time period is added is a period to original principal. The resulting amount becomes the new
principal for the next time period. Thus the interest earned in prior periods earns interest in the
future periods.

pg. 73 Prepared by: Firehun A


If at the end of a payment period the interest due is reinvested at the same rate, then the interest
n
as well as the original principal will earn interest during the next (1+i) payment period. Interest
paid on interest reinvested is called compound interest. If P is the principal earning interest
compounded m times a year at an annual rate of i for t years, then the future value, F, at the end
the period is given by the following definition.
Definition: Maturity value (Future Value):
F=P(1+i)n where P = the principal [present value];
i = interest rate per period;
n = total number of compounding periods; Note that:

1. n = m⋅t ,where m number of compounding periods per year; and t is the time
period in year
r
i=
2. m where r = annual nominal rate

Now, let me give the following example on compound interest. Please follow the steps of the
solutions so that you can do other similar problems by yourself.
Example: If Br. 2,000 is invested at 8% compounded
a. annually;
b. semi–annually; and
c. quarterly, what is the amount after 5 years?
Given P = Br. 2,000; r = 8% = 0.08; t = 5 years
a. i = 0.08; n = m⋅t = 1 ¿ 5 = 5

F=P(1+i)n
5
F = 2000(1+0 . 08) = 2000 ¿ 1.469328 = Br. 2938.66
5
[refer the Table 1* attached at the back pages of this module to see the value of (1+0 . 08) to
be 1.469328]
r 0 . 08
i= = =0 . 04
b. Semi-annually: m = 2: m 2 ; n = m⋅t = 2 ¿ 5 = 10.
10
F = 2000(1+0 . 04 ) = 2000 ¿ 1.480244 = Br. 2960.49

pg. 74 Prepared by: Firehun A


10
[refer the Table 1* attached at the back pages of this module to see the value of (1+0 . 04 )
to be 1.480244]
r 0 . 08
i= = =0 . 02
c. Quarterly: m = 4: m 4 ; n = m⋅t = 4 ¿ 5 = 20.
20
F = 2000(1+0 . 02) = 2000 ¿ 1.485947 = Br. 2971.89
20
[refer the Table 1* attached at the back pages of this module to see the value of (1+0 . 02)
1
10
to be 1.485947] [refer Table 2* to see the value of (1+0 . 04 ) to be 0.67556].

Example: If an investment company pays you 8 % compounded semi-annually, how much


should you deposit now to have Br. 10,000 after 5 years?
F = Br. 10,000; t = 5 years; and r = 8% are given. Thus, the present value, P, will be
calculated using:
F 10000
P= = =
(1+i )n (1+0 . 04 )10 10000 ¿ 0.67556 = Br. 6755.60
Hence, Br. 6755.60 must be deposited now in order to obtain Br. 10,000 after 5 years
compounded semi-annually at 8% compound interest.
Example: How long will it take Br. 4,000 to grow to Br. 9000 if it is invested at 12% rate of
interest compounded quarterly?
Principal = P = Br. 4000; F = Br. 9000; Nominal rate = r = 12% ;
Number of compounding period per year, m = 4;
Number of total compounding periods, n = m⋅t = 4⋅t ; Rate per period, i ,
12 %
=3 %=0 . 03
= 4
Thus solving for t, from the future value formula,

( )
mt
r
P 1+
F=P(1+i)n = m
it is possible to have,

pg. 75 Prepared by: Firehun A


log ( F / P)
t=
m⋅log 1+ ( r
m )
log (9000/4000 ) log2 . 25
=
Thus t = 12⋅log ( 1+0 . 03 ) log1 . 03
= 0.35218/0.01284
= 27.43 years or 27 years and 5 months
4.3.2. Nominal and Effective Interest rate
The effective annual interest rate is commonly abbreviated as the effective rate. If the principal is
Birr 1, the value of the compound interest for one-year period is the effective rate. Normally, the
effective rate is greater than the nominal rate. To obtain a relation for the effective rate, the
following assumptions are made. If a principal is invested at an annual rate, compounded m
m
times a year, then in one year: F = P ( 1+ i ) .
The simple interest rate will produce the same amount in one year; the simple interest rate is

called the effective rate, r e . To find r e , use the following relation.


Amount at simple interest after 1 year = Amount at compound interest after 1 year.

( )
m
r
P 1+
P(1+r e )= m

1+r = ( m)
m
r
1+
⇒ e , thus

[( ) ]
m
r
1+ −1
⇒ re = m that is known as the effective rate.

Definition:
If principal P is invested at the rate of r compounded m times a year,
then the effective rate re is given by

( ) −1
m
r
1+
re = m .

pg. 76 Prepared by: Firehun A


Example: Find the effective rate if money is worth 6% compounding quarterly on investment
market.
The nominal rate = r = 6%; the number of compounding period per year, m = 4;

[( ) ]
m
r
1+ −1
Effective rate, r e =
m

=
[( ) ]
1+
6% 4
4
−1
;
= 1.06136355 - 1 = 0.06136355 = 6.14%
This shows that money invested at 6.14% simple interest earns the same amount of interest in
one year, as money invested at 6% compounded quarterly is 6.14%.
Example: Bank-I offers its depositors an interest rate of 8% compounded quarterly, while Bank-
II gives its depositors an interest rate 9% compounded yearly. Which of the two banks makes the
better offer?
The effective rate based on the interest rate Bank-I is

[( ) ] [( ) ]
m
r 8% 4
1+ −1 1+ −1
re = m =
4 = 1.08243 – 1 = 0.08243 = 8.243%
4
[refer table 1* to find the value of( 1+2 % ) to be 1.08243 )
The effective rate based on the interest rate of Bank-II is

[( ) ] [( ) ]
m
r 8% 1
1+ −1 1+ −1
re = m =
1 = 1.082 – 1 = 0.082 = 8.2%.
The effective rate of Bank-I is greater than that of Bank-II by
8.243% - 8.2% = 0.043%
Therefore, Bank –I offers a better interest rate to its depositors.

pg. 77 Prepared by: Firehun A


4.4. Annuities
Dear distance learner, in this section you will study about ordinary annuity, which is the subject
of annuity that affects the life of practically every family in a society. One of the financing of
huge social security problem is a type of annuity problem.
An annuity is a sequence of payments, usually of equal size, made at periodic time interval. The
word Annuity implies yearly payments but the term implies to all periodic payments plans, the
most frequent of which require annual, semi-annual, quarterly or monthly payments. Practical
applications of annuities are widely encountered in the financial dealings of businesses and
individuals alike , payments or receipts of money relating to rent, leases, pensions, family
allowances, wages and salaries,. . . etc are example of annuities. Here I considered two sections
to present the two basic ideas that point towards to the present value of an ordinary annuity and
the future value of an annuity.

4.4.1. Future Value of an Ordinary Annuity


An annuity is any sequence of equal periodic payments. If payments are made at the end of each
time interval, then the annuity is called ordinary annuity. If payments are made at the beginning
of each time interval, then the annuity is called annuity due.m
The amount, future value, of an annuity is the sum of all payments plus all interest earned.

Definition:
Future value of an ordinary annuity, F, is defined by the formula

F=R [
( 1+ i )n−1
i ]
Where R = periodic payment; i = rate per period;
n = number of payments (periods)

Example: What is the value of an annuity at the end of 10 years if Br. 1,000 is deposited every 6
months into account that earn 8% rate of interest compounded semi-annually? How much of this
value is interest?
We have
n = (10 years)¿ (2 times per year) = 20 periods, which means 20 semi-annual payments.

pg. 78 Prepared by: Firehun A


r 8%
i= = =4 %
m 2
where m = number of compounding periods = 2 per year, and
r = nominal rate = 8%.

Period payments, R = Br. 1000


Thus, applying the definition given above gives

F=1000 [
( 1+ 0 .04 )20−1
0 . 04 ]
[
[refer to table 3* to see the value of
( 1+0 . 04 )20−1
0 . 04 ]
to be 29.77808]
Hence, F = 1000 ¿ 29.77808 = Br. 29,778.08
Therefore, the value of annuity is Br. 29,778.08, and
the interest = F – (R¿ n) = ( 29778.08 – 1000¿ 20) = Br. 9778.08
Example: Find the accumulated value of quarterly payments of Br. 50 at the end of each quarter
for 10 years just after the last payments has been made and if interest is 16% compounded
quarterly. How much of the value is interest?
We have
n = t⋅m = (10 years)¿ (4 times per year) = 20 periods, which means 40 quarterly payments.

r 16 %
i= = =4 %
m 4
where m = number of compounding periods = 4 per year, and
r = nominal rate = 16%.

Period payments, R = Br. 50


Thus, applying the definition given above gives

F=50 [
( 1+ 0 .04 )40−1
0. 04 ]
[
[refer to table 3* to see the value of
( 1+0 . 04 )40−1
0 . 04 ]
to be 95.02552]
Hence, F = 50¿ 95.02552 = Br. 4751.28
Therefore, the value of annuity is Br. 4751.28, and

pg. 79 Prepared by: Firehun A


the interest = F – (R¿ n) = (4751.28 – 50¿ 40) = Br. 2751.28

4.4.2. Present Value and sum of an ordinary amount


As discussed in the above sections, an annuity is a series of periodic payments, usually made in
equal amounts. The payments are computed by the compound interest method and are made at
equal intervals of time, such as annually, semi- annually, quarterly, etc,. The word annuity
originally referred only to annual payments, but now applies to payments interval of any length
of time. The period of time between two consecutive payment dates is called the payments
interval. The time between the beginning of the first payments interval and the end of the last
payments interval is called the term of the annuity.
The present value of an annuity is the value at the beginning of the term of annuity. You have
learned earlier that the present value of any amount due n periods from now is found by
multiplying the amount by the compound discount factor,

(1+i)−1

If we multiply the future amount of an annuity of Br. 1 per period, that is,
F=1 [ i ]
(1+i )n −1
,
by the compound discount factor we have the present value of an annuity of Br. 1 per period,
which is obtained as,

P= [ 1−( 1+ i )−n
i ]
Thus the present value, P, of an ordinary annuity of Br. R per period is then defined as:

Definition:
Present=value of ordinary annuity, P,

P=R [
1− (1+i )−n
i ]
where R = periodic payment; i = rate per period;
n = number of payments (periods)

pg. 80 Prepared by: Firehun A


Example: What sum deposited now in an account earning 8 percent interest compounded
quarterly will provide quarterly payments of Br. 1,000 for 10 years , the first payment to be made
3 months from now?
Here we have
n = (10 years)(4 quarter per year) = 40

8%
i= =2%
4

[ 1−( 1+ 2% )−40
2% ]= 27 .35548
[from Table 3*]
R = Br. 1000
Hence,
P = 1000(27.35548) = Br. 27355.48
Example: Find the present value of an annuity of Birr 150 payable at the end of each year for 15
years if interest rate 5% compounded annually?
Here we have
n = (15 years)(1 per year) = 15

5%
i= =5 %
1

[ 1−( 1+5 % )−15


5% ]=10 . 37966
[from Table 3*]
R = Br. 150
Hence,
P = 150(1037966) = Br. 1556.95
The present value of an annuity is, therefore, Br. 1556.95.
Example: How much should you deposit in an account paying 8% compounded quarterly in
order to receive quarterly payment of Br. 1000 for the next 4 years?
n = (4 years)(4 quarters per year) = 16

8%
i= =2%
4

[ 1−( 1+ 2% )−16
2% ]=13 . 57771
[from Table 3*]

pg. 81 Prepared by: Firehun A


R = Br. 1000
Hence,
P = 1000(13.57771) = Br. 13577.71 must be deposited now to pay Br. 1000 for the next 4 years.

1.4.3Annuity due
An annuity due is an annuity for which the periodic payments are made at the beginning of each
payments interval. The term of an annuity due begins on the date of the first payments and ends
one payments interval after the last payments is made. The amount of an annuity due is the value
at the end of the term of the annuity. It includes all the periodic payments plus the compound
interest. By the method similar to that used in finding the amount of an ordinary annuity, the
amount of an annuity due may be found by totaling the individual compound amounts of the
periodic payments.

A simpler method of finding the amount of an annuity due is to use the formula for finding the
future value of an ordinary annuity, F. When the formula is used, the amount of an annuity due
may be found as follows:
First find the amount of the Ordinary Annuity of (n + 1) payments. Then, subtract the additional
payments from the amount obtained. That is,

F=R [ i ]
( 1+ i )n+1 −1
−R
.
Example: What is the amount of an Annuity Due for one year if each payment is Br. 100
payable at the beginning of each quarter and the interest rate 4% compounded quarterly?
R = Br. 1000
n = (1 years)(4 quarters per year) = 4

4%
i= =1 %
4

[ ( 1+1 % )5−1
1% ]
=5. 10101
[from Table 5*]
Hence, F = [100¿ 5.10101] – 100; since payments are made at the beginning.
F = 510.10 – 100 = Br. 410.10,
which is the future value of an annuity if payments are made at the beginning of the periods.

pg. 82 Prepared by: Firehun A


4.4.3. Amortization and Sinking Fund
Dear distance learner, in many financial transactions a current obligation is discharged by
making a series of payments in the future. After the last payment, the obligation ceases to exist, it
is said to have been amortized by the payments. In this section, you will study about amortization
and sinking fund, and the techniques applied to the construction of their schedules. They are
important in repayment of loan and accumulation of sum of money for the fulfillment of some
obligations.
Since the Section is important in social life, I advise you to study it carefully and relate it to your
saving and bank payments to loans.
Amortization
Amortization refers to the repayment of interest- bearing debts by a series of payments, usually
equal in size, made at equal intervals of time. The periodic payments when equal in size form an
annuity whose present value is equivalent to the original loan principal. Mortgages and many
consumer loans are repaid by this method. The allocation of each payment first to cover the
interest due and then reduce the principal may be shown in an amortization schedule.

If both principal and interest are repaid by a series of equal payments made at equal intervals of
time, then an interest–bearing debt is said to be amortized. The basic problem in amortizing a
debt is finding the size of the periodic payment. If the payment interval and the interest
conversion period are equal in length, the problem involves finding the periodic payment for a
simple annuity.

Typical examples of amortization are loans taken to buy a car or a home and amortized over a
period of 20 to 30 years in the case of a home mortgage and over 2, 3, or 4 years in the case of a
car purchase loan. Given the amount of the loan (the current principal, P), the number of periods
(n), and the interest (i), the quantity to be calculated is R, the amount of the periodic payment.
The n payments of Br. R each constitute an ordinary annuity whose present value is P, and you

pg. 83 Prepared by: Firehun A


have learned that P is R times the present value of an ordinary annuity of Br. 1, which is

[ 1−(1+i)−n
i ]
. Therefore,

P=R
1−(1+i)−n
i [.
]
Solving this for the unknown R, you have
P
R=

[ 1−( 1+i)−n
i .
]
The denominator is a simple fraction, so we may invert it and multiply by the numerator to
obtain

Definition:
Amortization payment of an ordinary annuity, R,

R=P
[ i
1− (1+i )−n ]
Example: A man borrowed Br. 1,200 from a bank and agreed to make 12 equal monthly
payments at the end of every month for one year. If interest is 18% compounded monthly, what
is the size of the monthly periodic payments?
Given the present value, P = Br, 1,200; the number of payments per year = m = 12;
the time = t = 1 year; the number of payments = n = 12; and
18 % 1
i= =1 %
the rate per period of payments = 12 2

R=P
[ i
1− (1+i ) −n
=
1200
] [1 .5 %
1−( 1+1 .5 % )−12 ]
1. 5 %
[Refer to Table 4* to see the value of 1−( 1+1. 5 % )
[
−12 ]
= 0.091679952]
Hence, R = 1200 x 0.091679952 = Br. 110.02.

pg. 84 Prepared by: Firehun A


Example: If Ayelu borrows Br. 500 with the agreement to repay in 6 equal monthly payments at
1% interest per month on the unpaid balance, find the monthly amortization?
The present value, P = Br, 500;
the number of payments = n = 6; and
the rate per period of payments = i=1 %

R=P
[ i
] [
1− (1+i )−n =
500
1%
1−( 1+1 % )−6 ]
[ 1%
[Referring Table 4* gives the value of 1−( 1+ 1% )
−6 ]
= 0.172548]
Hence, R = 500 x 0.172548 = Br. 86.27.

Amortization Schedules
In the case of loans repaid in fixed installments, the constant periodic payment is first applied to
pay the accumulated interest. The remainder of the payment is then used to reduce the unpaid
balance of principal.
While lenders are obliged to disclose to the borrower the total cost of borrowing as well as the
effect of the periodic payments on the principal may be obtained by constructing a loan
repayment schedule, often referred to as an amortization schedules. The information usually
contained in such a schedule includes:
a. amount paid at each payment date;
b. the interest paid by each payment;
c. the principal repaid by each payment;
d. the unpaid loan balance after each payment.
Example: A debt of Br. 5,000 is amortized by making equal payments at the end of every three
months for two years. If interest is 15% compounded quarterly, construct an amortization
schedule.
Step_1: Determine the size of the quarterly payments. [I will advice you to use scientific
calculator];
R = Br. 5,000; number of compounding periods per year = m = 4;
total number of compounding period = n = 4 x 2 = 8;

pg. 85 Prepared by: Firehun A


15 %
i= =3 . 75 %
rate of interest = 15%; and rate per period, 4 .

R=P
[ i
1− (1+i ) ]
−n
=
5000
[ 0 .0375
1−( 1+0 . 0375 )−8 ]
[ 0 . 0375
Using calculator gives the value of 1−( 1+ 0 .0375 )
−8
to be
] [5000
x 0.146998383] = Br. 734.99.

Step_2: Construct the amortization schedule as shown below.


Outstanding
Payment Amount Interest Paid Principal
Principal
Number Paid i=0 . 0375 Repaid
Balance
5000.00
0
734.99 187.50 547.49 4452.51
1
734.99 166.97 568.02 3884.49
2
734.99 145.67 589.32 3295.17
3
734.99 123.57 611.42 2683.75
4
734.99 100.64 634.35 2049.40
5
734.99 76.85 658.14 1391.26
6
734.99 52.17 682.82 708.44
7
735.01 26.57 708.44 0.00
8

Total 5879.94 879.94 5000.00

Here I will give a brief explanation on the construction of the above sinking fund schedule.
 Payment number 0 is used to introduce the initial balance of the loan.
 The interest included in the first payment is 0.0375 x 5000 = 187.50. Since the amount paid
is Br. 734.99, the amount available for repayment of principal is 734.99 – 187.50 = Br.
547.49. Hence, the outstanding principal balance after the first payment is 5000 - 547.49 =
Br. 4452.51.

pg. 86 Prepared by: Firehun A


 The interest included in the payment is 0.0375 x 4452.51 = Br. 166.97. Since the amount
paid is Br. 734.99, the amount available for payment of principal is 734.99 - 166.97 = Br.
568.02.
Hence, the outstanding principal is 4452.51 -568.02 = Birr 3884.49.
Other computations were also being carried out in a similar manner.
 The last payment of Br. 735.01 is slightly different from the other payments as a result of
rounding in the amount paid or the interest paid. To allow for such rounding errors, the last
payment is computed by adding the interest due in the last payment ( 0.0375 x 708.44 =Br.
26.57) to the then outstanding balance. That is’
708.44 + 826.57 = Br. 735.01.
 The three totals shown provide very crucial information and can be used as a check for the
accuracy of the schedule.

Example: If Ayelu borrows Br. 500 with the agreement to repay in 6 equal monthly payments at
1% interest per month on the unpaid balance, construct a partial amortization schedule for the
first, the last interval periods and totals.
The present value, P = Br, 500;
the number of payments = n = 6; and
the rate per period of payments = i=1 %

R=P
[ i
1− (1+i ) ] [
−n
=
500
1%
1−( 1+1 % )−6 ]
[1%
[Referring Table 4* gives the value of 1−( 1+ 1% )
−6 ]
= 0.172548]
Hence, R = 500 x 0.172548 = Br. 86.27.

Now we can construct the amortization schedules as shown below.


Outstanding
Payment Interest Paid Principal
Amount paid Principal
Number i=0 . 0375 repaid
Balance
0 500.00

pg. 87 Prepared by: Firehun A


1 86.27 5.00 81.27 418.73
2 86.27 4.19 82.08 336.65
3 86.27 3.37 82.90 253.75
. . . . .
. . . . .
. . . . .
6 86.30 0.85 85.45 0.00
Total 517.65 17.65 500.00

Sinking Fund
Under the sinking fund method, it assumed that a sinking fund is established for the purpose of
replacing an asset at the end of its useful life. The periodic deprecation charges are exactly the
same as the periodic increase in the sinking fund. Thus, the depreciation charges for each year
are not equal. However, the total of the depreciation charges is equal to the amount in the sinking
fund at the useful life of the asset. The size of each deposit made in the sinking fund can be
obtained by using the annuity formula.

The sinking funds are interest–bearing funds into which payments are made at periodic time
intervals to provide a desired sum of money at a specified future point in time. Such funds
usually involve large sums of money used by both the private sector and the public sector repay
loans, redeem bonds, finance future capital acquisition, provide for the replacement of
depreciable plant, equipment and recover investments in delectable natural resources. The basic
problem in dealing with sinking funds is that of determining the sizes of the periodic payments
which will accumulate to a known future amount. These payments form an annuity in which the
accumulated value is known.

Depending on whether the periodic payments are made at the end or at the beginning of each
payment period, the annuity formed is an ordinary annuity or an annuity due. Depending on
whether or not the payment interval is equal in length to the interest conversion period, the
annuity formed is a simple annuity. However, since sinking funds are equal in length, only the

pg. 88 Prepared by: Firehun A


ordinary annuity cases are considered. For sinking fund payments with payments at the end of
each payment interval is defined as a fund in to which periodic payments are made in order to
accumulate a specified amount at some point in the future. In order to determine the required
periodic payment, R, in to a sinking fund, it possible to use the n payments that constitutes an
ordinary annuity of R per period, and the known future value of this annuity
( 1+i )n −1
F=R
i

Solving this for the periodic payment R, one can generate

[ ]
1
R=F
(1+i)n −1
i
Inverting the simple fraction in the denominator, and multiplying this by the numerator, 1, it is
possible to get sinking fund formula, which is given as follows.

Definition:
Sinking Fund Payments, R

R=F
[ i
(1+i)n −1 ]
Example: How much should be deposited in a sinking fund at the end of each quarter for 5 years
to accumulate Br. 10,000 if the fund earns 8 percent compounded quarterly?
8%
i= =2%
4
n= (5 years)(4 quarters per year) = 20 periods. This shows

i
( 1+i )n −1 = 0.0411567.
Hence,

pg. 89 Prepared by: Firehun A


R = 10,000(0.0411567) = Br. 411.567.
Thus the quarterly payment is rounded, Br. 411.57. Over the life of the sinking fund, the sum of
the deposits will be
20(411.567) = Br. 8,231.34.
This sum plus interest earned will provide the desired Br. 10,000.
Example: A sinking fund of Br. 20,000 is to be created by making equal deposits at the end of
every six months for four years. Interest is 12% compounded semi-annually.
The size of the semi annual deposit into the fund will be obtained using the formula given in the
above definition as:
12 %
i= =6 %=0. 06
F = Br. 20,000; n = 4 ¿ 2 = 8; 2

R=20000
[ 0 .06
]=
20000
(1+0 .06 )8 −1 9 . 8974679 = Br. 2020.72

The total amount deposited into the fund is equal to


8 ¿ 2020.72 = Br. 16165.76
The amount of interest in the fund
20000 – 16165.76 = Br. 3834.24

Sinking Fund Schedules


The details regarding a sinking fund may also be presented in the form of a schedule. Sinking
fund schedules normally show the payment number, the periodic payment into the fund, the
interest earned by the fund, the increase in the fund and the accumulated balance.
Example: Let me construct a sinking fund schedules that refers to the above example.
R = Br. 2020.72; n = 8; i = 6% = 0.06
The sinking fund schedule is then obtained to be:
Increase for
Balance in Fund at
payment Increase
Payment the End of
Periodic
Interval Interval Period in Fund
Payment Payment interval
Number
i = 0.06
0 0.00

pg. 90 Prepared by: Firehun A


1 2020.72 0.00 2020.72 2020.72
2 2020.72 121.24 2141.96 4162.68
3 2020.72 249.76 2270.48 6233.16
4 2020.72 385.99 2406.71 8839.87
5 2020.72 530.39 2551.11 11390.98
6 2020.72 683.46 2704.18 14095.16
7 2020.72 845.71 2866.43 16961.59
8 2020.72 1071.70 3038.42 20000.00
Total 3834.25 20000.00

Here I will give a brief explanation on the construction of the above sinking fund schedule.
 The payment number 0 may be used to introduce the beginning balance.
 The first deposit is made at the end of the first payment interval. Hence, the interest earned
by the fund during the first payment interval is Br. 2020.72; the increased in the fund is Br.
2020.72 and the balance is Br. 2020.72.
 The second deposit is added at the end of the second payment interval. The interest for the
interval is 0.06 ¿ 2020.72 = Br. 121.24; hence, the increase in the fund is 2020.72 + 121.24
= Br. 2141.96 and new balance in the fund is 2020.72 + 2141.96 = Br4162.68.
 The third deposit is made at the end of the third payment interval. The interest for the interval
is 0.06 ¿ 4162.68 = Br. 249.76; the increase in the fund is 2020.72 + 249.76 = Br. 2270.48
and new balance in the fund is 2270.48 + 4162.68 = Br. 6433.16.
Similar calculations were carried out to fill the remaining payment intervals.
 The final balance in the sinking fund will likely be slightly different than the expected value.
This is a result of rounding the decimal numbers appeared after the decimal point in two digit
figures.
 The three totals shown are very crucial and should be obtained for each schedule. The total
increase in the fund must be the same as the final balance.

Example: The XYZ company decides to establish a building fund of Br. 130,000 by making
equal deposits at the end of every three months in to a sinking fund for seven years. Interest is
12% compounded quarterly.

pg. 91 Prepared by: Firehun A


a. Compute the increase in the fund during the 12th payment interval.
b. Construct a partial sinking fund schedule showing details of the first three deposits, the 12 th
deposit, and the last three deposit and total.
12 %
i= =3 %
F = Br. 130,000; n = 7 ¿ 4 = 28; 4 . Thus,

R=F
[ i
]
(1+i)n −1
=130000
[ 3%
=
]
(1+3 % )28−1 Br. 3028.12
a. Balance in the fund at the end of the 11th payment interval
F 11=3028 . 12×12 .807796= Br. 38,783.54

The interest earned by the fund during the 12th payment interval is
0 . 03×38783 . 54= Br. 1163.51
The increase in the fund during the 12th payment interval is
1163.51 + 3028.12 = Br. 4191.63
b. The last three payments are payments 26, 27, and 28. To show this one has to develop the
accumulated value after 25 payment intervals.
F 11=3028 . 12×36 . 459264= Br. 110,403.03

Thus, the partial sinking fund schedule is obtained as:

Interest paid Increase Outstanding


Payment Periodic
i = 0.06 in Fund Principal Balance
Number Payment
0 0.00
1 3028.12 0.00 3028.12 3028.12
2 3028.12 90.84 3118.96 6147.08
3 3028.12 184.41 3212.53 9359.61
. . . . .
. . . . 38783.54
12 3028.12 1163.51 4191.63 42975.17
. . . . .

pg. 92 Prepared by: Firehun A


. . . . 110403.03
26 3028.12 3312.09 5340.21 116743.24
27 3028.12 3502.30 6530.42 123273.66
28 3028.12 3698.21 6726.33 129999.99
Total 45212.63 129999.99

UNIT - 5
Elements and Application of Calculus (10)
5. Basic Concepts of Limits
In this section, you will learn basic concepts of limits and show you that the concept of limits is
very crucial to the study of calculus. This concept helps you to describe, in a precise way, the

behavior of the function f ( x ) as x approach a particular value in the set of real numbers. This is
a base for the two major sections of calculus, namely Differentiation and Integration.
Definition of Limit a function of one variable
Limits are the core concept in the development of calculus. In this section you will become
familiar enough with the limit of concept to use it when it is needed. Let start with the function
x2
f ( x )=
x
This function does not have a value at x = 0, because at this value of x the ratio is the
meaningless expression0:0, and we say the function f is not defined at x = 0. It is true, though,
x2
f ( x )= =x , x≠0
that x

That is, f ( x )=x for any value of x except zero. Because this is true, it is correct to say that f ( x)

is close to zero if x is close to zero or that f ( x ) approaches zero if x approaches zero. We

describe this behavior of f ( x ) by saying that f ( x ) approaches zero as a limiting value as x

approaches zero and that as x goes to zero, the limit of f ( x ) is zero. This is expressed
symbolically as

pg. 93 Prepared by: Firehun A


lim f ( x )
¿
x →0 ¿
where the symbol “ x → 0 ” means x approaches zero or gets closer and closer to zero without

ever equaling zero. Thus, in the case of f ( x ) we are able to state that a limit exists. It is
lim f ( x )
¿
especially important to note that x →0 ¿ is not always related to the value of f ( x ) at x = 0.
lim f ( x )
¿
In this particular example, f (0) does not exist but x →0 ¿ .
By way of contrast, consider
x
g( x )= ; x ≠0
x2

like the earlier f ( x ) , g( x) at x = 0 becomes the meaningless expression 0:0 and g( x) is not
defined at x = 0. It is true, of course, that
x 1
g( x )= = , x≠0
x2 x
1
g( x )=
That is x for any value except x = 0. But g( x) , unlike, f ( x ) , does not have a limit as x
approaches zero. For example, if x takes on the sequence of values
1, 0.1, 0.01, 0.001,
and so on, approaching zero, then
1
g(1 )= =1
1
1
g(1 )= =10
0. 1
1
g(1 )= =100
0. 01
1
g(1 )= =1000
0. 001
and so on, and this last sequence of values become larger and larger and approaches ∞ as x gets
closer and closer to zero. Similarly if we took x = -1, -0.1, -0.01, -0.001, and so on, as a sequence
approaching zero, then

pg. 94 Prepared by: Firehun A


1
g(1 )= =−1
−1
1
g(1 )= =−10
−0. 1
1
g(1 )= =−100
−0. 01
1
g(1 )= =−1000
−0. 001
and so on. Again the sequence -1, -10, -100, -1000 continues indefinitely and approaches -∞ .

Hence,
x
lim 2
¿
x →0 x
¿ does not exist.
Comparing
x2
f ( x )= , x≠0
x , and
x
g( x )= , x≠0
x2
we see that they are alike at x = 0 in that both becomes the meaningless ratio, 0:0. However, they

have the important difference that f ( x ) has a limit as x approaches zero and g( x) does not. The
distinction is important, because, as we shall see, the central definition of differential calculus
always leads to the ratio 0:0, but we can attach a meaningful interpretation to the expression that
leads to the ratio 0:0 when this expression has a limit.

Definition: Informal Definition of Limit


lim f ( x )
¿
x→c ¿ ;

if the functional value f ( x ) is close to the single real number L


whenever x is close to but not equal to c, on either side of c.

pg. 95 Prepared by: Firehun A


lim ( x+3)
¿
Example: Find x→2 ¿ keeping in mind that x → 2 means x must not equal 2, we set up
any sequence of x values approaching 2, and compute the corresponding values of the function x
+ 3. For example,

x 1.9 1.99 1.999 1.9999 → 2


f ( x )=x +3 4.9 4.99 4.999 4.9999 → ?
or
x 1.5 2.25 1.875 2.0625 → 2
f ( x )=x +3 4.5 5.25 4.875 5.0625 → ?

All indications seem to be that the “?” at the right of the sequence for f ( x )=x +3 is 5, so that
we conclude
lim ( x +3 )
¿
x →2 ¿
x 2 −4
lim
x−2 ¿
x →2 ¿
Example: Find, if it exits,
A graph of the function is again useful. First we see that the function is not defined for x = 2
since the denominator would be zero. Next we see that if x≠2 , then x – 2 is not zero, so we can
x 2 −4
lim
x −2 ¿ =
lim ( x+2 )
x →2 ¿
cancel x – 2 with a factor of x 2
−4 ; that is ¿ x→2 ¿
We emphasize the factor that we can perform the above cancellation because the value of x is
never equal to 2 when we let x approaches 2, hence x – 2 is never zero in the above limit. Thus,
x2 −4
the graph of x−2 must look just like the graph of x + 2 everywhere except at x = 2, as shown
in Figure 5-1.
Now it is easy to see that
x 2 −4
lim
x−2 ¿=
lim ( x+2 )
x →2 ¿
¿ x→2 ¿ =4

pg. 96 Prepared by: Firehun A


Figure 2-1
Example: Not all functions are as easy to handle as the above examples.
Let’s try another one. Let

f ( x )= {−11 ifif x≥2


x <2
lim f (x )
¿
What is the x→2 ¿ ?

Now observe the following sequences:


x 3 2.5 2.25 2.125 → 2
f ( x) 1 1 1 1 → 1
or
x 1 1.5 1.75 1.875 → 2
f ( x) -1 -1 -1 -1 → -1
In the first instance,
lim f ( x )
¿
x →2 ¿ as x approaches 2 from the right. But in the second instance,
lim f ( x )
¿
x →2 ¿ as x approaches 2 from the left. For
lim f (x )
¿
x→2 ¿ to exist, we must have

pg. 97 Prepared by: Firehun A


lim f ( x ) lim f ( x) lim f (x )
¿ ¿ ¿
x→2+ ¿ = x→2− ¿ = x→2 ¿
The only logical conclusion to be drawn in this case is that
lim f (x )
¿
x→2 ¿
does not exit.
x
lim ¿
x →2 x−2
Example: Find, if it exist, ¿
When working with functions, it is often helpful to draw a graph, as shown in Figure 5-2.
Utilizing the new-found significance of left-hand-side and right-hand-side limits, let us check
x
lim
x−2 ¿
x →2
¿
as x approaches 2 from right and from left.

Figure 2-2
To check the right-hand-limit, we can use the sequence in the following table:
x 3 2.5 2.25 2.125 → 2
+
f ( x) 3 5 9 17 →

We conclude that

pg. 98 Prepared by: Firehun A


x
lim ¿
x → 2 x− 2
¿
x
lim
x− 2 ¿
x →2
Chose a sequence to show that ¿ .
Using either of the sequences, we conclude that
x
lim ¿
x →2 x−2
¿
does not exist.

We now state and illustrate limit theorems we shall refer to from time to time as our work
progresses. These theorems are intuitively reasonable, but their proof requires attention to
details.
Limit Theorems:
lim k
¿
1. If k is any constant, x→a ¿
Here note that x → a specific that x is changing and approaching a. The constant k does not
change. Thus the limit of a constant is a constant.
lim 10
¿
Example: x → 4 ¿
lim kf ( x) lim f ( x)
¿ k ¿
2. x→a ¿ = x→ a ¿
That is, a constant factor, here k, may be placed inside or out side the limit symbol.

lim 3 x4 ¿
Example: x → a ¿
lim [ f ( x )± g ( x ) ] ¿
3. x→ a ¿
That is, the limit of a sum or a difference is the sum or difference of the limits. Or, we can
say that the limit of an expression can be taken term by term.

pg. 99 Prepared by: Firehun A


lim [ x 2 −2 x +3 ] ¿
Example: x → a ¿
lim [ f ( x )( x ) ] ¿
4. x→ a ¿
That is, the limit of a product is the product of the limits.
lim [ ( x +3 )( x −2 ) ]
¿
Example: x → 3 ¿
= [6][-1]
= -6

lim [ f ( x ) ] n ¿
5. x→ a ¿
That is, the limit of a power of f ( x ) is the power of the limit of f ( x ) .

lim [ x −1 ] 3 ¿
Example: x → 3 ¿
lim f ( x)=L lim g(x )=M
¿ ¿
6. If x→a ¿ and x→a ¿ , then

lim
x→a
[ ]
f ( x) L
= ;
g( x ) M ¿
a. if M ≠0 , then ¿

lim
f ( x)
x→a g( x )
¿ [ ]
b. if M =0 and L≠0 , then ¿ does not exist;

c. if M =0 and L=0 , then f ( x ) and g( x) have a common factor and the limit can be
evaluated after employing the process of cancellation.

7. If f and g are functions that have limits as x approaches c and f ( x ) = g( x) for all x≠c , then
lim f ( x )
¿
x→ c ¿
Now let us proceed to the informal definition of the limit of a function f as x approaches to ∞ or
-∞ .

pg. 100 Prepared by: Firehun A


Definition: Informal Definition of Limit at Infinity
lim f ( x)=L
¿
1. x→∞ ¿ ; if the functional value f ( x ) is close to the single
real number L whenever x is very large positive real number.

lim f ( x)=M
¿
2. x→∞ ¿ ; if the functional f ( x ) is close to the single real
number M whenever x is very small negative real number.

5 x +4
lim ¿
x→∞ 2 x−3
Example: Find, if it exists ¿
5 x +4
Since it does not change the magnitude of the expression 2 x−3 if we divide both the numerator
and denominator by x, we can equally express the above limit as:

[ ]
4
5+
x
=
3
2−
lim x
¿
x→ ∞

¿
lim
x →∞( 5 + 4x ) = 5 + 0 = 5
x → ∞( x )
lim 3 2 +0 2
2−
, and hence as x approaches to ∞, the function
5
value f ( x ) approaches to 2 .
Continuity
It is often useful to consider sets of functions with a common property. Continuity is such a
property. In simple terms, continuous functions are functions with graphs that are unbroken. Let
us sketch a graph of

f ( x )= {x+1, if x≥0
x−1 , if x <0
as shown in Figure 5-3.

pg. 101 Prepared by: Firehun A


Figure 2-3
This graph is fragmented in that we cannot draw the graph in one continuous stroke without
having to lift our pencil from the paper. Roughly speaking, a function is called continuous if its
graph can be drawn without lifting the pencil from the paper. In what circumstances do we have
to lift the pencil from the paper while drawing?

As we see in Figure 2-3 and 2-4, if there is a jump in the graph, or, as in Figure 2-5, a gap or
hole in the graph, at x = a, we then must lift the pencil. In the first instance, the jump in Figure 2-
lim f ( x)
¿
4, we notice that x→a ¿ does not exit at the jump point a since the right-hand limit does not
equal the left-hand limit. Think about this a moment and it will become clear that to avoid a
lim f ( x)
¿
jump at a point a, x→a ¿ must exit.

In the second instance, the gap in Figure 5-5, the function f ( x ) does not have a value at x = a.

Clearly f ( x ) must have a value f (a ) if we are to draw the graph in one continuous motion. Are
these two conditions enough? Before you answer let us look at one more special case in Figure
lim f ( x) lim f ( x )
¿ ¿
5-6. In this graph, x→a ¿ exists and f (a ) is defined, but x→a ¿ f (a ) .

pg. 102 Prepared by: Firehun A


Figure 2-4

Figure 2-5

Figure 2-6

pg. 103 Prepared by: Firehun A


Now, with all this in mind, we are ready for a formal definition of continuity.

Definition:

A function f ( x ) is said to be continuous at a point a if the following


three conditions are met:

1. f (a ) is defined.
lim f ( x)
¿
2. x→a ¿ exists.
lim f ( x)=f (a)
¿
3. x→a ¿ .
If a function is not continuous at a point a we say it is discontinuous
at a or has a discontinuity at a.

Example: Discuss the continuity of


x +2
f ( x )= 2
x −3 x +2
First, we factor the denominator
x +2
f ( x )=
( x−1 )( x−2)
Now we can go through the three conditions of the definition to check for points of discontinuity.
Once we have found these points of discontinuity, if any, then the function will clearly be
continuous at all other points.

Condition 1: f ( x ) is defined for x except x = 1 and x = 2.


lim f ( x)
¿
Condition 2: By limit theorem 6, we see that x→a ¿ exists for x except x = 1 and x = 2.
lim f ( x)=f (a)
¿
Condition 3: Again by limit theorem 6, we see x→a ¿ for x except x = 1 and x = 2.

Hence, f ( x ) is continuous at all x except x = 1 and x = 2, where it has discontinuous.

pg. 104 Prepared by: Firehun A


The concept of continuity and discontinuity can be simplified by recognizing that certain classes
of functions are always continuous. For our purposes, the following list, presented without
justification, will be very helpful in determining the continuity of functions.
1. All polynomial functions are continuous. A polynomial function is a function that can be
reduced in to the form
a n x n +a n−1 x n−1 +an−2 x n−2 +¿ ⋅⋅ +a1 x+a0 , where a 0 , a1 , a2 , ⋅⋅⋅, an ∈ ℜ and n ∈ Z .

2. If f ( x ) is continuous and g( x) is continuous, then

a. f ( x )±g( x ) is continuous.

b. f ( x )⋅g( x ) is continuous.

c. f ( x)/ g( x) is continuous when g( x )≠0

5.2. Derivatives and Rules of Differentiation


You now have the tools needed to obtain the rules of differential calculus. In this section you will
learn about derivatives and rules of differentiation of functions. The section is divided into three
subsections. Here I give you different rules of differentiation which are useful in solving
important problems, which are related to business activities, like optimization of profit and costs.

The Derivative of a Function


The notion of the derivative, alongside with notion of the integral, is the most important concept
in mathematical analysis. The derivative of a function f at a point x is defined as the limit to

which tends the ratio of the increment Δy of the function at that point to the corresponding
increment Δx of the argument as Δx tends to zero.
The derivative of a function f is usually defined in the following way:

Definition:

For y = f ( x ) the derivative of the function f at x, denoted by f ( x ) , is


'

defined as:
Δy
'
f ( x )= lim ¿
Δx → 0 Δx
¿ if the limit exists.
'
pg. 105 If f ( x ) exits, then f is said to be a differentiable function at Prepared
x. by: Firehun A
df ( x ) dy
' ,
Other notations such as y , dx dx are also widely used. The convenience of this of that
notation will be appreciated afterward by the reader himself. The function just stated is the
fundamental definition of differential calculus, and all rules for finding derivatives are developed
by starting from this definition. To compute the derivative of a function f is to differentiate f, and
the process of obtaining the derivative of a function is called differentiation.

Note that if you substitute a specific value for x in the definition,


f ( x +Δx )−f ( x )
f ' ( x )=lim Δx ¿
Δx→0
¿ ,
'
you obtain f ( x ) = slope of the tangent line to the graph of y = f ( x ) at that substituted point.

Definition:
The slope of a curve at a point means the slope of the line tangent to
the curve at that point.

' 2
Example: Find the derivative of f at x, f ( x ) , for f ( x )=x .
f ( x +Δx )−f ( x ) ( x+ Δx )2−x 2
' lim lim
f ( x )= Δx ¿ Δx ¿
Δx→0 Δx→0
¿ = ¿

lim [ x +2 xΔx+ Δx ]−x


2 2 2

= Δx ¿
Δx→0
¿
2 xΔx+ Δx2
lim ¿
Δx→0 Δx
= ¿
lim [ 2 x+Δx ]
¿
= Δx→0 ¿

pg. 106 Prepared by: Firehun A


= 2x
'
By substituting various specific values for x in the derivative f ( x ) = 2x, you can obtain the slope
2
of the tangent to the graph f ( x )=x at various points. For example, the slope of the tangent line
2 ' '
to the graph of f ( x )=x at x =2, x = 3 and x = 2 are f ( 2 )=2×2=4 , f ( 3 )=2×3=6 , and
f ' ( 0)=2×0=0 , respectively.

Example: Find the derivative of f ( x )=√ x .


f ( x +Δx )−f ( x )
' lim
f ( x )= Δx ¿
Δx→0
¿

lim √
x +Δx−√ x
Δx ¿
Δx→0
= ¿

lim
Δx→0
[ Δx ][
√ x + Δx−√ x × √ x +Δx+ √ x
√ x +Δx+ √ x ¿ ]
= ¿ rationalizing the numerator
x +Δx−x
lim
¿
Δx→0 Δx ( √ x +Δx+ Δx )
= ¿
1
lim
¿
Δx→0 ( √ x +Δx+ √ x )
= ¿
1 1
=
= ( √ x +√ x ) 2 √ x
1
f ' ( x )=
Therefore, the derivative of f ( x )=√ x is 2√x
2
Find the slope of f ( x )=x +1 at x = -1, 0 and 1. [Use the space given below to write your
response]
Basic Rules of Derivatives
Derivative of a Constant Function

pg. 107 Prepared by: Firehun A


Consider the constant function, f, defined by f ( x )=c . Here for every x there corresponds one
and the same value y = c. Hence, the derivative of this function is obtained to be:
f ( x + Δx )− f ( x )
' lim
f ( x )= Δx ¿
Δx → 0
¿
That is, the derivative of a constant function is zero.
Derivative of a Power Function [Power Rule]
n
Consider the Power function, f, defined by f ( x )=x for every real number n. Then the
derivative of this function is given by:
'
f ( x )=nx n−1
5 5−1
Example: The derivative of f ( x )=x is obtained to be f ( x )=5 x =5 x 4 .
Remark:
n ' n−1
Suppose k is a constant and n is any real number. If g( x )=kx , then g ( x )=knx
' 3
Example: Find f ( x ) for f ( x )=5 x .
'
f ( x )=5×3 x 3−1 =15 x 2
Derivatives of Sum and Differences of Functions [Sum and Difference Rule]
Let f be a function defined as a sum or difference of two component functions u and v. That is,
f ( x)=u( x )±v( x ). Then f ' ( x )=u' ( x )±v ' ( x ). In other words, the derivative of a sum is the sum
of the derivatives, and the derivative of the difference is the difference of the derivatives. Note
that this rule is applicable to compute the derivatives of all polynomial functions.
' 3 2
Example: Find f ( x ) for f ( x )=5 x + 4 x +1 .
′ ′
f ' ( x )=( 5 x 3 ) + ( 4 x 2 ) + ( 1 )′ =15 x 2 +8 x +0
Derivatives of Products of Functions [Product Rule]
Let f be a function defined as a product of two component functions u and v. That is,
f ( x)=u( x )×v( x ). Then f ' ( x )=u' ( x )v ( x )+u( x )v ' ( x ).
' 2
Example: Find f ( x ) for f ( x )=(2 x +3 )×(2 x −2 x ) .

f ( x )=(2 x+3) ×(2 x 2 −2 x )+(2 x+3 )×(2 x 2 −2 x )


' ' '

pg. 108 Prepared by: Firehun A


2
=(2)×(2 x −2 x )+(2 x +3)×( 4 x−2)
2
= 12 x +4 x −6
Derivatives of Quotient of Functions [Quotient Rule]
Let f be a function defined as a product of two component functions u and v. That is,
u( x ) ' '
u ( x ) v ( x )−u( x ) v ( x )
f ( x )= . f ' ( x )= 2
.
v ( x Then [v( x)]
( 2 x+ 3)
f ( x )=
Example: Find f ( x ) for
'
( 2 x 2−2 x ) .
' '
(2 x +3 ) ×(2 x 2−2 x )+(2 x +3 )×(2 x 2−2 x )
f ' ( x )=
(2 x 2−2 x )2
(2)×(2 x 2−2 x )+(2 x +3 )×( 4 x−2)
= (2 x 2 −2 x )2

12 x2 +4 x −6
2 2
= (2 x −2 x )

Derivatives of Composite Function [Chain Rule]

If f and g are two functions, the function that assigns to every x the value f (g ( x)) is called the
composition of f and g and is denoted by fog. For example, if we are given the two functions

f ( x )=sin x and g( x )=x2 , then the composition function f (g ( x ))=( fog ) ( x )=sin ( x 2 ) .

The derivative of the composite function, ( fog ) (x ) = f ( g (x ))×g ( x )
' '

' 2 100
Example: Find f ( x ) for f ( x )=(1+x ) .
2
Let g( x )=1+ x . Thus, now you have;
2 100 ′
f ( x )= ( fog ) (x ) = f ( g (x ))×g ( x ) = ( (1+ x )
' ′ ' '
) ( 1+x 2 )′
= [ 100(1+ x ) ] [ 2 x ]
2 99

2 99
= 200 x (1+ x )
Remark:

pg. 109 Prepared by: Firehun A


k
Let f is a power of a function defined by the equation f ( x )=[ g( x ) ] . Then
k −1 '
f ( x )=k [ g( x ) ]
'
g (x )
5.2.1. Applications of Derivatives
In this Section you will study how to apply the techniques of differential calculus to solve
optimization problems and sketch curves. I will begin by illustrating maxima and minima of
simple polynomial functions and develop the concept of critical points along with the first
derivative test. Next we will use concavity to introduce the second derivative test and illustrate
how to use this test in many different applications.

For your study convenience, I have divided the Section into two subsections: first and second
derivatives of functions and applications of first and second derivatives tests.
5.2.2. Marginal Analysis
The word marginal is the economist’s terminology that stands for the rate of change in cost,
profit, or revenue. For example, the additional cost incurred with certain additional amount of
items produced. Therefore, the marginal analysis is concerned with the rate at which one
economic variable changes relative to another. Do you remember that you have learnt about the
three linear functions which are appreciated by manufacturers; the cost function, the revenue
function and the profit functions? If all units of items produced are sold, the three functions are
related by:
p( x)=r( x)−c( x) .
The derivatives of these three functions are called the marginal cost, the marginal revenue, and
the marginal profit, respectively.
That is,
c ' ( x )= Marginal Cost
= Rate of change of cost with respect to the number x of units produced.
'
r ( x )= Marginal Revenue
= Rate of change of revenue with respect to the number x of units produced.
p' ( x )= Marginal Profit
= Rate of change of profit with respect to the number x of units produced

pg. 110 Prepared by: Firehun A


Applying the Sum and Difference rule of derivative will help us in developing the relationship
between the derivatives of the above stated functions as:
' ' '
p ( x )=r ( x )−c ( x ) . That is, MP = MR + MC.
Example: Suppose that the manufacturer’s cost and revenue functions are given by the
equations:
x2
c ( x )=3000+200 x+
5 , and

x2
r ( x )=350 x+
20 .

Where c( x) and r( x) are in Br. Construct the equations that will define the marginal cost,
marginal revenue, and marginal profit and determine these marginal values if 20 units of items
are produced and sold by the manufacturer.
' 2x
c ( x )=200+
Marginal Cost = 5
' x
r ( x )=350+
Marginal Revenue = 10

' '
Marginal Profit = p ( x )=r ( x )−c ( x ) =
' [ 200+
2x
5 - ][
350+
x
10 ]
=
[ 150−
]
3x
10
If x = 20, then
2(20)
c ' (20)=200+ =Br . 208
5
Which means that the cost is increasing at a unit of Br. 208 per unit at the time the sales level
reaches x = 20.
' 20
r (20 )=350+ =Br . 352
10 , and
3(20 )
p' (20)=150− =Br . 144
10

5.2.3. Optimization Problem

pg. 111 Prepared by: Firehun A


One of the most useful applications of calculus in management and economics is finding the
maximum and minimum values for a function. Many practical problems reduce to maximizing or
minimizing some quantity: a business person wants to maximize the profits and minimize the
costs, an engineer wants to maximize the strength of a bridge structure, and an airliner wants to
minimize the flying time for a certain trip and so forth. In this part you will learn how
differentiation can be used to solve certain kinds of maximization and minimization problems.
Let me start with an example, just to illustrate the concepts of optimization.
Example: A farmer wants to fence in rectangular field bordering on a straight on a stream, but
will not fence the side along the stream. If there are 1000 feet of fence to work with, what is the
maximum area can be enclosed?
y

x x
Stream

To solve this problems, let us denote the length of the sides by x and y, so that the area A to be
maximized is A = x ¿ y.
Since the farmer has 1000 feet of fence, we have the following relationship between x and y:
1000 = 2x + y, or y = 1000 - 2x.
2
Thus we can rewrite as A = x ¿ (1000 - 2x) = 1000x - 2 x .
In this formula the variable x is subject to certain physical restrictions. For example, since x
represents a length, we must have x ¿ 0. Moreover, there are only 1000 feet of fence available,
so we cannot use more than this amount on the two sides of length x; thus 2x ¿ 1000 or x ¿ 500.

2
In light of these physical restrictions and A = 1000x - 2 x , we can formulate our problems. The
physical problems of fencing a field with maximum area reduced to the mathematical problem of
2
maximizing. A = 1000x - 2 x , where x is required by physical considerations to lie in the closed
interval [0, 500].
This is typical characteristics of many optimization problems: the objectives is to maximize or
minimize some function f (x), where x is required to lie in some specified interval.

pg. 112 Prepared by: Firehun A


Definition:
1. On a specified interval, a function f is said to have an absolute maximum
at x = c if f (c) is the largest value of f (x) for every x in the interval,
similarly, f has an absolute minimum at x = c if f (c) is the smallest value
of f (x) for every x in the interval.
2. If a function f is continuous at each point of a closed interval [a, b], then f
has an absolute maximum ad an absolute minimum on [a, b].

If f is differentiable at each point of a closed interval [a, b], then there are two possibilities for
the location of an absolute maximum; it can occur at one of the end-points a, or b, or it can occur
at a point where the derivative of f is zero. This possibility holds true for the location of an
absolute minimum. Note that, a point in the interval at which the derivative of f becomes zero is
called the critical value.

Definition: If f is differentiable at each point of given interval and if f has


an absolute maximum on the interval, then this maximum must occur at one
end-point or at a critical value; It also holds true for an absolute minimum.
Note that, in the case of maximizing or minimizing a continuous function over
a closed interval [a, b], you need not worry about the existence of this
maximum or minimum.

Basic procedures for finding on absolute maximum or minimum of a differentiable function over
a closed interval [a, b]:
a. Find the critical values of f on [a, b].

pg. 113 Prepared by: Firehun A


b. Evaluate f at the critical values and end-points a, and b.
c. The largest of the values obtained in [b] is the absolute maximum for f, and the smallest is
the absolute minimum.
Note that, when you work on the maximum or minimum values of f over a non-closed intervals,
you must worry about the possibility that the maximum or minimum may exist.
Example: Find the absolute minimum and maximum of the function defined by

f ( x )=2 x 3−3 x 2 −36 x+5 On the interval [-5, 5].


'
a. You can find the critical values of f on [a, b] by setting f ( x )=0 .
' 2
First obtain the derivative of the function; that is, f ( x )=6 x −6 x −36
'
Then setting f ( x )=0 , you have
6 x 2 −6 x−36=0 , and solving the resulting quadratic equation yields,
x = -2 and x = 3, which are the critical values of the function.
b. Evaluate f at the critical values and at the end-points. This leads us to have:
f at critical values

f (−2 )=2(−2)3 −3 (−2 )2−36 (−2)+5=49


f (3)=2(3)3 −3(3 )2 −36(3 )+5=−76
f at end-points

f (−5 )=2(−5 )3−3(−5 )2 −36(−5)+5=−140


f (5)=2(5)3 −3(5 )2−36 (5)+5=0
c. Hence, the absolute minimum value of f is [-140] and it occurs at x = -5; similarly, the
absolute maximum value of f is [49] and it occurs at x = -2.
First and Second Derivative Tests for Maxima and Minima
In this section I will give additional mathematical tools, which facilitates things in solving
maximization and minimization problems.

'
At points where f ( x )> 0 , the curve y = f (x) has a tangent line with positive slope, and at points
'
where f ( x )< 0 , the curve has a tangent line with negative slope. Thus it follows that the curve y
'
= f (x) will be increasing on any interval where f ( x )> 0 for all x in the interval and decreasing

pg. 114 Prepared by: Firehun A


' '
on any interval where f ( x )< 0 for all x in the interval. Recall that a value of x where f ( x )=0
is called a critical value of f. At such points, the curve y = f (x) is neither rising nor falling.
2
Example: Let f ( x )=x −2 x +6 . Where is y = f (x) increasing? Decreasing? What are the
critical values for f?
'
Since f ( x )=2 x −2 , the critical values of f occur when 2x – 2 = 0.
Solving for x yields x = 1 as the only critical value.
' ' '
Since you can write f ( x )=2 x +2=2( x−2) , you see that f ( x )> 0 if x > 1 and f ( x )< 0 if x <
1.

Figure 8
3 2
Example: Let f ( x )=2 x −3 x −12 x +2 . Where is y = f (x) increasing? decreasing? What are
the critical values of f?
' 2 2
Since f ( x )=6 x −6 x −12=6( x −x−2 )=6( x−2 )( x+1) , the critical values of f occur when
6(x - 2)(x + 1) = 0.
Thus, the critical values are x = 2 and x = -1.

'
Since f ( x )=6( x−2 )( x+ 1) , the sign chart for 6(x-2)(x+1) is

pg. 115 Prepared by: Firehun A


Interval x <−1 x=−1 −1< x <2 x=2 x >2

Sign of x +1 - 0 + + +

Sign of x - 2 - - - 0 +

Sign of 6 (x −2)( x +1) + 0 - 0 +

Increasing Decreasing Increasing


' '
You have f ( x )> 0 when the factor x – 2 and x + 1 have the same sign, and you have f ( x )< 0
when the sigh are opposite. From the analysis in the figure below, you see that y = f (x) is
increasing on the interval (-∞ , -1) and (2, ∞) and decreasing on the interval (-1, 2). The graph
of f is show below.

Figure 9
If peak occurs at a critical value of f, then f is said to have a relative maximum or local maximum
at the point; and if a trough occurs, then f is said to have a relative minimum or local minimum at
the point.
The relative maximum or minimum may or may not be an absolute maximum or minimum of a
function. And the relative minimum and maximum of a function can be identified by the
behavior of the derivative in the vicinity of the point. At a relative maximum the graph of f (in
the above Example) is increasing on an interval extending left from the point and decreasing on
an interval extending right from the point; at a relation minimum, the graph is decreasing on the
left and increasing on the right.

pg. 116 Prepared by: Firehun A


1. The First Test for Maximum and Minimum
' '
If f ( x ) exists on both sides of a critical value c, then the sign of f ( x ) can be used to determine
if the point (c, f (c)) is a local maximum, a local minimum, or neither.

Definition: First Derivative Test

a. f '
has a relative maximum at c if f ( x ) < 0 on an interval extending left
'
from c and f ( x ) < 0 on an interval extending right from c.

b. f '
has a relative minimum at c if f ( x ) < 0 an interval extending left from
'
c and f ( x ) > 0 on an interval extending right from c.

c. f has neither a relative maximum nor a relative minimum at c if


'
f (x)
> 0 on an interval extending left from c and on an interval extending right
'
from c or f ( x ) < 0 on such intervals.
'
To paraphrase this result loosely, a relative maximum occurs if the sign of f changes from + to
'
– at x, a relative minimum occurs if the sign of f changes from - to + at c, and neither occurs if
'
f does not show any change of sign at c.
We can also use our knowledge of the increasing-decreasing behavior of a function to determine
the nature of endpoint optima.

Definition: End point Test


If has an end point value at x = p, then
' '
a. If p is a right-hand endpoint and f ( x ) < 0 ( f ( x ) > 0) to the left of p so that f ( x)
is decreasing down (increasing up) to f ( p ) , then f ( x ) has an endpoint minimum
(maximum) at x = p;
b. If p is a left-hand endpoint and f ( x ) < 0 ( f ( x ) > 0) to the right of p so that f ( x )
' '

is increasing up (decreasing down) to f ( p ) , then f ( x ) has an endpoint maximum


(minimum) at x = p.

pg. 117 Prepared by: Firehun A


The following graph illustrates both the above tests.

Figure 11
Now, let me give you examples to illustrate the given definitions and new concepts.
1 1
f ( x )= x 3 − x 2
Example: Given 3 2 .
a. Find the critical values of f.
b. Find the local maxima and local minima.
' 2
a. f ( x )=x −x= x( x−1 )
⇒ x (x - 1) = 0
⇒ x = 0 or x - 1 = 0
⇒ x = 0 or x = 1.

The critical values are x = 0 and x = 1.


'
b. The easiest way to apply the first–derivative test is to construct a sign chart for f ( x )
for all x.
' 2
Sign chart for f ( x )=x −x= x( x−1 )

pg. 118 Prepared by: Firehun A


Interval x <0 x=0 0< x <1 x=1 x >1

Sign of x - 0 + + +

Sign of (x – 1) - - - 0 +

Sign of x ( x−1) + 0 - 0 +

Local Local
Increasing Decreasing Increasing
maxima minima

The sign chart indicates that f increases on (−∞, 0) , has a local maxima at x = 0 and maximum
1 1
f (0)= (0 )3 − (0 )2=0
value is 3 2 . It decreases on (0, 1) and has a local minimum at x = 1 and
1 1 1
f (1)= (1)3 − (1 )2=−
the minimum values is 3 2 6
3 2
Example2: Given f ( x )=2 x −3 x −12 x +2
a. Find the critical values of f.
b. Find the local maxima, local minima, the maximum and the minimum values.

' 2
a. f ( x )=6 x −6 x −12=6( x−2)( x+1)

⇒ 6 (x −2)( x +1)=0
⇒ x = 2 or x = -1.

The critical values are x = 0 and x = 1.


'
b. The easiest way to apply the first–derivative test is to construct a sign chart for f ( x )
for every x.

pg. 119 Prepared by: Firehun A


' 2
Sign chart for f ( x )=6 x −6 x −12

Interval x <−1 x=−1 −1< x <2 x=2 x >2

Sign of x +1 - 0 + + +

Sign of x - 2 - - - 0 +

Sign of 6 (x −2)( x +1) + 0 - 0 +

Local Local
Increasing Decreasing Increasing
maxima minima

The sign chart indicates that f increases on(−∞, −1) , has a local maxima at x = -1 and maximum
3 2
value is f (−1 )=2(−1) −3(−1) −12(−1)+2=9 . It decreases on (-1, 2) and has a local
3 2
minimum at x = 2 and the minimum values is f (2)=2(2) −3(2) −12(2 )+2=−18
Example: A manufacturer has the following costs in producing x toasters in one day, 0 < x <
150: fixed costs, $320; unit production cost $20 per toaster; equipment maintenance and repairs,
x2
$20 . Thus, the cost of manufacturing x toasters in one day is given by:
x2
c ( x )= +20 x+ 320; 0< x <150
20
x 320
c ( x )= +20+ ; 0< x <150
And the average cost per toaster is given 20 x . Find the critical
values forc( x) , the interval where the average cost per toaster is decreasing, the intervals where
the average cost per toaster is increasing, and local extrema.

x 320
c ( x )= +20+ ; 0< x <150
20 x
1 320
c ′ ( x )= − 2 =0
20 x
2
⇒x −320(20)=x 2−6400=0 .

pg. 120 Prepared by: Firehun A


⇒ ( x−80 )( x+80)=0 .

Thus, the critical value of ( x) on the interval (0, 150) is x = 80.
c
The sign chart for c(x) is:

Interval x=0 x <80 x=80 x >80

Sign of x - 80 - - 0 +

Local
c ′ ( x) Decreasing
minima
Increasing

Therefore, c ′ ( x) is increasing for x > 80 and decreasing for 0 < x < 80; c ′ ( x) has a local
minimum x = 80.
2. The second Derivative Test for Maximum and Minimum
Now, let me show you how the second derivative can be used to test critical values to determine
whether they correspond to relative maxima or to relative minima. Assume that c is a critical
value of f. If f’ (c ) > 0, then the curve y =f(x) is concave up about c indicating a relative minima
at c. On the other hand, if f’( c)< 0, then y = f(x) is concave down at c indicating a relative
maximum at c. This suggests the following result.

The Second Derivative Test


'
1. Find f ( x ) , set it equal to zero, and solve for candidate values, x.
'' ''
2. Find f ( x ) and valuate f ( x ) .
''
a. If f ( x ) is negative, a local maximum occurs at x.
''
b. If f ( x ) is positive, a local minimum occurs at x.
''
c. If f ( x ) is zero, the test fails to determine what happens.
Example: Find the local maxima and minima of the following function. Use the second
derivative test when it applies.
1 1
f ( x )= x 3 − x 2
3 2 .

pg. 121 Prepared by: Firehun A


' 2
You have f ( x )=x −x= x( x−1 )
⇒ x (x - 1) = 0
⇒ x = 0 or x - 1 = 0
⇒ x = 0 or x = 1.

The critical values are x = 0 and x = 1.


'' ''
Since f ( x )=2 x−1 ,we have f (0 ) = 2(0) - 1 = -1 < 0.
⇒ There is a relative maximum at x = 0, and
''
since f (1) = 2(1) - 1 = 1 > 0, there is a relative minimum at x = 1. This agrees with the results
obtained in the above example using the first derivative.
''
Please note that the second derivative test does not specify the nature of the critical value, f ( x )
= 0.
Steps in finding absolute maximum and minimum values of continuous functions:
1. Check whether the given function, f, is continuous over [a, b];
2. Find the critical values in the interval (a, b);
3. Evaluate f at the end points and the critical values found step 2;
4. The absolute maximum of f (x) on [a, b] is the largest of the values found in step 3; and
5. The absolute minimum of f (x) on [a, b] is the smallest of the values found in step 3.
Example:
Find the absolute maximum and absolute minimum values of

f ( x )=x 3 +3 x 2 −9 x−7 on the interval [-6, 4]


The function is continuous for all values of x with in the given interval.
'
f ( x )=3 x 2 +6 x−9 = 3(x - 1)(x + 3).
Thus, x = -3 and x = 1 are critical values in the interval (-6, 4). Evaluate f at the end points and
critical values, -6, -3, 1, and 4, and choose the maximum and minimum from these.
f (−6) = -61 Absolute minimum
f (−3 ) = 20
f (1) = -12
f (4 ) = 69 Absolute maximum

pg. 122 Prepared by: Firehun A


Basic strategies for solving applied optimization problems
Step 1: Introduce variable and construct a mathematical model of the form:
Maximize or minimize f (x) on the interval I.
Step 2: Find the absolute maximum or minimum value of f (x) on the interval I and the values of
x where this occurs.
Step 3: Use the solution to the mathematical model to answer the questions asked in the
application.
Example: The marketing research department for a computer company used a large city to test
marketing their new product. They found that a relationship between price p dollars per unit and

the demand x units per week was given approximately by p=1296−0 . 12 x 2 , 0 < x < 80.
Thus, the weekly revenue can be approximated by

r( x )=xp=1296 x −0 .12 x 3 , 0 < x < 80.


a. Find the local extreme for the revenue function.
b. Over which interval is the graph of the revenue function concave upward?
Concave downward?

r( x )=1296 x−0 .12 x 3 , 0 < x < 80.


'
r ( x )=1296−0 . 36 x 2
Critical values:

r ( x )=1296−0 . 36 x 2 = 0
'

1296
x 2=
⇒ 0. 36
⇒ x = ± 60

Thus, x = 60 is the only critical value on the interval (0, 80).


''
r (x )=−072 x
''
⇒r (60)=−0 .72(60 )=−43 .2<0

pg. 123 Prepared by: Firehun A


''
Therefore, r has a local maximum at x = 60. Since r (x )=−072 x < 0 for every x on the interval
(0, 80) r is concave downward on this interval

5.3. Integration
5.3.1. Concept of integration
Dear distance learner, you have learned how to find the derivatives of functions and that
numerous applications of the derivative stem from its interpretation as the slope of the curve

representing the function. It quite surprising to learn that if we start with a function, f ( x ) , carry
out the process that is the inverse of taking the derivative, the result provides an area that has

f ( x ) as part of its boundary. The inverse process is first called taking the anti-derivative of f ( x )

then, later, we shall refer it as integrating f ( x ) .

Many of the applications of the inverse process stem from interpreting the result as an area that
represents quantities such as Br. of profit. Traditionally, Calculus is divided into two main areas:
Differential Calculus, which is concerned with tangents to curves and their applications, and
Integral Calculus, Which is concerned with areas under curves and their applications.
In the first part of this section, emphasis is placed on practicing integrating different functions;
then attention turns to areas having these functions as part of their boundaries.
5.3.2. Anti- derivatives: Indefinite Integrals and
Definite Integrals
Addition and subtraction are examples of inverse operations where one operation annuls the
effect of the other. Thus, if we start with number 50, add 10 to it, then subtract 10, we have the
original number 50. Similarly, multiplication and division are inverse operations and, as a last
example, cubing a number is annulled by the inverse operation of taking the cube root. In similar
'
fashion here you will learn the inverse operation that will guide us in finding f given f .

pg. 124 Prepared by: Firehun A


Now recall the rule for the operation of taking the derivative of x to a power n, that is, multiply
n
the function x by the power n and subtract one from the power to obtain a new power, n – 1,
thus getting
d n
( x )=n⋅x n−1
dx
Keeping this rule in mind, then, correct inverse procedure would be to add one to the power n – 1
to obtain a new power n and subsequently divide the entire expression by this new power to get
n
the original function x , that is,

n⋅x (n−1)+1 n⋅x n n


= =x
(n−1)+1 n
If we take
d 3
( x ) =3 x 2
dx
and attempt to reverse the process of differentiation, it is possible to obtain
x 2+1 3 x 3
= =x 3
2+ 1 3 ,
as desired. However, the correct procedure just shown is still incomplete, because if we have
d 3
( x +10 )=3 x 2
dx ,
2
and apply the above procedure to 3 x , we can not recover the constant 10. What we can say is
2 3
that 3 x the derivative of x plus an arbitrary constant, C, and we write
2 3
anti-derivative (3 x ) = x + C.
Note that, the process of obtaining a function from its derivative is called anti-derivative or
integration.
For reasons that will become clear when we introduce interpretations of the anti-derivative
d
operation, we shall represent this operation by an elongated S just as the symbol dx represents

the operation of differentiation; thus, ∫ is called the integral symbol and means the anti-
derivative of the expression following, as in

pg. 125 Prepared by: Firehun A


3
∫ x 3 dx= x3 +C . ∫ f ( x)dx=F ( x)+C . where F ( x) is anti-derivative of
In general, we write
f ( x ).

Note that, ∫ f ( x)dx is called the indefinite integral of f (x). The arbitrary constant C is called the
constant of integration, and the function f (x) is called integrand. Since integration is the reverse
of the differentiation, every differentiation formula has a companion integration formula. For
example, if K is a constant then we can define some of them.

5.3.3. Rules (Properties )of the integration operation

These properties parallel those of the derivative operations. First, note that the derivative of x is

1, so that

∫ 1 dx =x+C .
Similarly, we know that

d
(3 x )=3
dx so that

∫ 3 dx=3 x +c
and, in general, for any constant k,

∫ k dx=kx+c
In the same way we can define other properties of the integration operations as follows:

x k +1
∫ x k dx= k +1
+c

 ∫ kf ( x) dx=k∫ f ( x) dx

pg. 126 Prepared by: Firehun A


 ∫ [ f ( x)±g(x )] dx=∫ f (x ) dx±∫ g( x) dx
1
∫ x dx=ln|x|+ C

Now let me use the above stated properties to evaluate the following indefinite integration.
Example: Evaluate each of the following indefinite integrals

a. ∫ 4 dx=4 x+C
b. ∫ dx=x+C
x9
∫ 3 x 8 dx=3∫ x8 dx=3⋅ 9
+C
c.
2 4
∫ [ 2 x 3−4 x 2+1 ] dx=2∫ [ x 3 ] dx−4 ∫ [ x 2 ] dx+∫ [ 1 ] dx= 4 x 4− 3 x 3+x +C
d.
2
e. The marginal cost in Br. is given by m( x )=x +3 x . Find the cost function c( x) , assuming
that the fixed cost (cost when the level of production x = 0) is Br. 30.
2
Since the marginal cost m( x )=x +3 x is the derivative of the cost function, we have

c ( x )=x 2 +3 x .
'

' 2
Thus the unknown cost function c( x) is an antiderivative ofc ( x )=x +3 x ; consequently, it can
be determined by integration:
3
x x2
c ( x )=∫ [ x 2 + 3 x ] dx=∫ [ x 2 ] dx+3 ∫ [ x ] dx= +3 + k
3 2
1 3
⇒ c ( x )= x 3 + x 2 + k
3 2
1 3
c ( 0 )= 03 + 02 +k ⇒k =30
Since c (0 )=30 , we have 3 2 .
1 3
c ( x )= x 3 + x 2 +30
Thus, the cost is 3 2 .
The Definite Integral
In differential calculus, you learned that, in geometric terms, the derivative of a function
evaluated at a point is the slope of the line tangent to the curve at that point. In this section, I will

pg. 127 Prepared by: Firehun A


show you that a form of the integral, in geometric terms, represents an area having the integrand
function as one of its boundaries.

For example, let there be given a non-negative continuous function f ( x ) on a closed interval [a,
b] (a and b are finite numbers). Its graph is represented in Figure 1. Let us set the following
problem: it is required to determine the notion of the area of a figure bounded by the curve y =
f ( x ) , the x-axis and two straight lines x=a and x=b , and to complete this area. It is natural to
solve this problem in the following way.

Let us divide the interval [ a , b ] in to n parts (subintervals) by points of division


a=x 0 < x 1 < x2 <¿ ⋅⋅¿ x i- 1 < xi < ¿ ⋅⋅¿ x n =b , [1]

On choosing an arbitrary points z i in each interval


[ x i-1 , x i ] (i=1, 2, ⋅⋅⋅, n ) [2]

Figure 12
( z i ∈[ x i-1 , x i ]) and evaluating the function f at these points, we form the sum
n
Sn =∑ f ( z i ) Δx i ( Δx i=x i−x i-1 )
i =1 [3]
which is called the integral sum and which is, obviously, equal to the sum of the areas of the
hatched rectangles [see Figure 13].

Let us now make all Δx i tend to zero so that the greatest partition subinterval tends to zero. If in

doing so Sn tends to a definite limit S which is independent of the ways of partition [1] and the

pg. 128 Prepared by: Firehun A


choice of points z i in the sub intervals, then it is natural to call S the area of our curvilinear
figure. Hence,
n

S= lim
∑ f ( z i ) Δxi ¿
max Δx i →0 i=1 ¿ .
Thus we have defined the area of our curvilinear figure. There arises the question whether each

such figure has an area, or in other words, its integral sum Sn tends, in fact, to a finite limit as
Δx i → 0 . It will be proved later on that this question is answered in the affirmative.

Definition 4:
Let f be a continuous function defined on the interval [a, b]. Divide
b−a
Δx=
the interval [a, b] in to n subintervals of equal length n

Choose points z 1 , z 2 , z 3 , ⋅⋅⋅ , z n such that


a=x 0 < z 1 < x1 < z 2 < x 2 < z 3 < x3 <¿ ⋅⋅¿ x n−1 < z n < x n=b
and form the sum
n
∑ f ( z i ) Δx
i=1

Called the integral sum. If the limit of the integral sum


n

lim ∑ f ( z i ) Δx ¿
n → ∞ i=1 ¿
exists and is equal to the real number I is called the definite integral
of f over the interval [a, b] and is denoted by
b

pg. 129
∫ f ( x ) dx Prepared by: Firehun A
I= a

and is read as “the integral of f with respect to x from a to b” and we


Remarks:
n

lim ∑ f ( z i ) Δx ¿
b
∫ f ( x ) dx n → ∞ i=1 ¿
1. In short, a =
b
∫ f ( x ) dx
2. From the above definition the value of the definite integral a depends upon the
function f and the lower and upper limits of integration a and b. The choice of the letter x has
no significance whatsoever. We could have equally expressed it for instance as:
b b b
∫ f (u ) du ∫ f (t ) dt ∫ f ( s) ds
a ,a , a , etc.
b
∫ f ( x ) dx
3. If f is continuous and f ( x )>0 in [a, b], the definite integral a represents the area of
the region S bounded by the graph of f, the vertical lines x=a , x=b and the x-axis (see
Figure 14).

Figure 13
It is not quite evident from the definition of the definite integral whether the limit of the integral
sum
n

lim ∑ f ( z i ) Δx ¿
n → ∞ i=1
¿
exists for all functions.

pg. 130 Prepared by: Firehun A


Theorem: The Fundamental Theorem of Calculus
If f is continuous on [a, b] and if F is an anti-derivative (indefinite-
'
integral) of f, that is, F ( x )=f ( x ) , for all x ∈ [a, b], then
b
∫ f ( x ) dx=F (b )−F (a )
a

3
∫ x dx
Example: Evaluate 1

'
The fundamental theorem of calculus asserts that if we can find a function F such that F ( x )=x ,
then the definite integral can be evaluated easily as F (3) – F (1).
x2
Now we know F (x) = 2 is an anti-derivative of f (x) = x.
Therefore, the Fundamental Theorem of Calculus says that
3
∫ x dx=F (3 )−F(1 )
1

32 12
= 2 - 2
8
= 2 =4

Example: Find the area of the region bounded by y=x 2 , x = 1, x = 3 and the x-axis.
We first graph the function and outline the desired area in Figure 14
The area is the definite integral of f (x) from 1 to 3.

x3 3
∫ x 2
dx=
3
|1
Area =
33 1 26
− =
= 3 3 3

pg. 131 Prepared by: Firehun A


Figure 14
26
Thus, the area is 3 square units.
2
Example: Find the area of the region bounded by f ( x )=x +8 x−12 , x = 3, x = 6, and the x-
axis. See Figure 15

Figure 15
We must find
6
−x 3
∫ (−x 2+8 x−12) dx= 3
+4 x 2 −12 x |63
3

= (-72 + 144 – 72) - (-9 + 36 - 36)


=9
The area is 9 squares unit.

pg. 132 Prepared by: Firehun A


b
∫ f ( x) d ( x)
If the region lies entirely below the x-axis, a will be a negative number. (Think of the
relationship between A(x), which represents area and is non negative, and f (x), which is
negative). In this case, the area can be computed by taking the absolute value of integral.

3
Example: Find the area of the region bounded by f (x) = x , x = -3, x = -1 and the x-axis.
The region is pictured in Figure 16.

Figure 16
−1
x 4 −1 1 81
∫ x 3 dx= |
4 −3 = 4 − 4 =−20
−3

Since |−20| = 20, the area of the region is 20 square units.

We are now ready to list some general properties of definite integral. Many of these properties
are inherited from those of indefinite integrals. These properties are listed in the following
Theorem. Suppose that all of the integrals listed in the theorem exist.

pg. 133 Prepared by: Firehun A


Theorem:
If f and g are integrable over [a, b] and k is any constant, then k, k f,
f + g and f - g are integrable over [a, b] and
b
∫ k dx=k (b−a )
1. a

b b
∫ k f ( x ) dx =k ∫ f ( x ) dx
2. a a

b b b
∫ [ f ( x )+g( x )] dx=∫ f ( x )dx +∫ g( x )dx
2. a a a

b b b
∫ (f ( x )−g( x ))dx=∫ f ( x )dx−∫ g( x )dx
3. a a a

b a
∫ f ( x ) dx=−∫ f ( x ) dx
4. a b

a
∫ f ( x ) dx=0
5. a

b c b
∫ f ( x ) dx=∫ f ( x ) dx +∫ f ( x ) dx
7. a a c

Example: Evaluate:
4
∫ 5 x 2 dx
a. 0

3
∫ (2 x +4 ) dx
b. −2

x n+1
n
a. Since n+1 is an anti-derivative of the function x for n ≠ -1 and by the above Theorem
property (1) of definite integral we have
4 4
∫5 x 5
dx =5∫ x 5 dx=5[ F( 4 )−F (0)],
0 0

pg. 134 Prepared by: Firehun A


x3
where F ( x ) = 2
3 is an anti-derivative of f ( x )=x

[
43 03
=5 3

2 ]
320
= 3
x n+1
∫ x dx = n+1 for n≠−1
n

b. By using and properties (1) and (2) of the above Theorem, we


have
3 3 3
∫ (2 x +4 ) dx=∫ 2 x dx +∫ 4 dx
−2 −2 −2 Property [2]
3 3
=2 ∫ x dx+4 ∫ dx
−2 −2 Property [1]

=2
2

2 (
3 2 (−2)2
) x2
+4(3−(−2)) [since ∫ x dx = and
2
∫ 1 dx=x ]
= 5 + 20
= 25
Example: A large factory on the Awash River discharge pollutant into the river at a rate that is

estimated by a water quality control agency to be p' (t )=t √t 2 +1 0 ¿ < t ¿ 5;

If p(t ) is the total number of tones of pollutants discharged into the river after 1 year of
operation, then what quantity of pollutants will be discharged into the river during the first 3
years of operation?

This example can be solved by substitution. First find an antiderivative and then find the net
change in that antiderivative.

[ ] dt
3 3 1
p(3 )− p(0 )=∫ [ t √t +1 ] dt =∫ t(t +1 )
2 2 2

0 0

pg. 135 Prepared by: Firehun A


[ ]
3 1
1
∫ 2
2 t(t +1 ) 2 dt
=2 0
3
3 3
3
2 2
1 (t +1) 1 2 2
| = (t +1) |
2 3 3
0
= 2 0
3
1
( 10 2 −1)
= 3
= 10.2 tons.

5.4. Application of Integration


5.4.1. Finding Total function
n

lim ∑ f ( z i ) Δx ¿
n → ∞ i=1 ¿
So far it has been dealt with the a limit of the form
arises when we compute an area. It turns out that this same type of limit occurs in a wide variety
of situations even when f is not necessarily a positive function. Note that limits of the form
shown above also arises in finding lengths of curves, volume of solids, areas of surfaces, centers
of mass, fluid pressure, work, acceleration, velocity as well as other quantities.
The integral has many interesting applications to geometrical and physical problems. In this
lesson I illustrate some of the application of the definite integral by using it to compute areas
between curves, marginal analysis etc.
5.4.2. Area Problem
Area between two curves
In previous section, we found the area of the region bounded by the curves y = f (x), the
x-axis, x = a and x = b to be the definite integral
b
∫ f ( x ) dx
a

We now wish to find the area between two curves y = f (x) and y = g(x) on the interval [a, b].
Assuming that both f and g are continuous and f lies entirely above g on this interval, we can

pg. 136 Prepared by: Firehun A


compute the area between them by subtracting the area under g from the area under f (see Figure
17).

Figure 17
Both the area under f and the area under g between x = a and x = b can be computed using
definite integral. Thus we have
b b
∫ f ( x ) dx−∫ g (x ) dx
Area between f and g = a a

b b
∫ f ( x ) dx−∫ g (x ) dx
= a a

This formula will work as long as f (x)¿ g( x ) for all x in the interval [ a, b] . Variations of this
formula can be used if such is not the case. In some problems, it may be your job to figure out
which figure is “on top’’ and which one is “on the bottom”.

Example: Find the area between f ( x )=x 2 + 4 and g( x )=1−x between x = -2 and x = 1.

Making a sketch of this region [Figure 18] we have


1
∫ [ f ( x )−g (x )] dx
Area = −2

1
∫ [( x2 +4 )−(1−x )] dx
= −2
1
∫ ( x 2+ x+ 3) dx
= −2

pg. 137 Prepared by: Firehun A


[ ]
1
x3 x2
+ +3 x
= 3 2 −2

(= 13 + 12 +3)−( −83 +2−6) = 212


Figure 18
21
The area between f and g is 2 square units.

2
Example: Find the area between f (x) = x + 2 and g(x) = −x between x = 0 and x = 2. Making
a sketch (Figure 19), we see that part of the area is below the x-axis the above formula still
works.
2
∫ [ f ( x )−g (x )] dx
Area = 0

2
∫ [ ( x+2)−(−x 2 )]] dx
= 0

= 3 [
x3 x2
+ + 2x
2 ]
(
8
= 3
)
+2+4 −( 0+ 0+0 )=
26
3

Figure 19

pg. 138 Prepared by: Firehun A


5.4.3. Application of Marginal Analysis
The marginal cost (MC), Marginal Revenue (MR) and the Marginal profit (MP) are the
derivatives of the cost function, revenue function and the profit function respectively. That is,
' '
MC = Marginal Cost = c ( x ) ; MR = Marginal Revenue = r ( x ) ; and MP =Marginal Profit =
'
p ( x ).
Thus, if you integrate MC, MR or MP over an interval [a, b] you obtain
b
∫ c' ( x ) dx=C (b )−C (a )
a = Change in cost as x varies from a to b.
b
∫ r ' ( x ) dx =R (b )−R(a )
a = change in revenue as x varies from a to b.
b
∫ p' ( x ) dx=P (b)−P(a )
a = Change in profit as x varies from a to b.
Example: A manufacture determines that the firm’s marginal cost and revenue functions are MC
' '
= c ( x )=100−0 .1 x ; RM =r ( x )=100+ 0 .1 x .
a. Find the revenue that results when the sales level increases from 20 to 30 units.
b. Find the revenue resulting from sale of 30 units.
c. If the fixed cost is Br. 400, then find the cost of producing 30 units.
a. As the sales level increases from x = 20 to x = 30, the revenue changes an amount
b 30
30
∫ r ( x ) dx =∫ [ 100+ 0 .1 x ] dx =[ 100 x +0 . 05 x 2]20
'

a 20

= 3045 – 2020
= Br. 1025
Thus the revenue increases by Br. 1025.
b. If you that the revenue from the sale of x = 0 units is zero, then the revenue resulting from the
sale of x = 30. This change is given by

pg. 139 Prepared by: Firehun A


b 30
30
∫ r ( x ) dx =∫ [ 100+0 .1 x ] dx =[ 100 x +0 . 05 x 2]0
'

a 0

= 3045 – 0
= Br. 3045
Therefore, the revenue received from the sell of 30 units of items Br. 3045 .
c. At the production level increase from x= 0- x =30, the cost changes an amount
b 30
30
∫ c ( x ) dx=∫ [ 100−0 .1 x ] dx=[ 100 x−0 . 05 x 2 ]0
'

a 0

= 2955 – 0
= Br. 2955
Thus, the total cost for manufacturing 30 units will be the fixed cost plus the
Added cost as the production increases form x = 0 to x = 30 units; that is
Total cost = Br. 400 + Br. 2955
= Br. 3355.

pg. 140 Prepared by: Firehun A

You might also like